Anda di halaman 1dari 57

BANKING AWARENESS

WITH SPECIAL REFERENCE TO BANKING INDUSTRY

Q.1. In which year Punjab National Bank was established ?


(1) 1880 (2) 1887 (3) 1894 (4) 1900 (5) None of these
Q.2. What is the full form of 'CORE' in core banking services ?
(1) Channel of Rupee Exchange.
(2) Customer Online Realtime Exchange.
(3) Centralized Online Rupee Exchange.
(4) Centralized Online Realtime Exchange.
(5) Customer Online Rupee Exchange.
Q.3. Consider the following statements about Multidimensional Poverty Index (MPI)
(A) MPI focuses on education, health & living standard as a supplement to income based
measure of poverty
(B) India ranks 63rd in MPI based poverty ranking among 104 developing countries.
(C) Bihar is the second poorest state in India on the basis of MPI based poverty ranking.
Which of above statement(s) is/ are true ?
(1) Only A & B (2) Only B & C (3) Only A & C (4) All A,B & C (5) None of these
Q.4. What is the upper limit of RTGS transactions in India ?
(1) No limit (2) Rs. 10 lacs (3) Rs. 50 lacs (4) Rs.20 lacs (5) Rs. 40 lacs
Q.5. Open unemployment refers to people-
(1) Who are not willing to work
(2) Who are willing but do not get work.
(3) Who leave their jobs in search of better jobs.
(4) Who have been dismissed because of corrupt practices.
(5) None of these
Q.6. 'Open Market Operation' is a part of-
(1) Income Policy (2) Fiscal Policy (3) Credit Policy
(4) Labour Policy (5) None of these
Q.7. SDR, the currency of the IMF, is in the form of-
(1) Paper Currency (2) Gold (3) Silver and Gold both
(4) Book Keeping entry only (5) None of these
Q.8. Which of the statements given below is/are incorrect ?
(A) United Stock Exchange is the fourth country level exchange of India.
(B) It is headquartered at Mumbai.
(C) It will trade in six foreign currency derivatives.
(1) Only A and B (2) Only B and C (3) Only A and C
(4) Only B (5) Only C
Q.9. In India, National income is estimated by ?
(1) Planning commission (2) Indian Statistical Institute
(3) Central Statistical Office (4) National Smaple Survey Organisation
(5) None of these

Mahendra's GENERAL AWARENESS


Q.10. Hindu Rate of Growth refers to the rate of growth of-
(1) GDP (2) Population (3) Foodgrains
(4) Per Capita income (5) None of these
Q.11. Which one of the following is not an objective of fiscal policy of Indian Government ?
(1) Full employment (2) Regulation of Inter-State-trade
(3) Price stability (4) Equitable distribution of wealth & income
(5) None of these
Q.12. How many times has financial emergency been declared in India so far ?
(1) 5 times (2) 4 times (3) Once (4) Never (5) None of these
Q.13. Which one of the following is not charged on the Consolidated Fund of India ?
(1) Salary & allowances of the president of India
(2) Salary & allowances of the speaker of the Lok Sabha
(3) Salary & allowances of the Justices of the Supreme Court of India
(4) Salary & allowances of the Vice President of India
(5) None of these
Q.14. What would be the New Price Index that would measure inflation ?
(1) PPI (2011) (2) CPI (2010) (3) WPI (2004-2005)
(4) RTI (5) None of these
Q.15. What is the maximum ceiling on Foreign Direct Investment (FDI) for investment in the equity of
Public Sector Banks in India ?
(1) 20% (2) 26% (3) 49% (4) 51% (5) 74%
Q.16. According to 2011 Census, Sex Ratio in India stood at -----females per 1000 males.
(1) 933 (2) 965 (3) 947 (4) 940 (5) None of these
Q.17. Indian Stock Market deals in --------.
(1) Shares (2) Debentures (3) Forward Contracts
(4) Mutual Funds (5) All of the above
Q.18. Self Help Group is associated with which of the following Rural Development Programmes ?
(1) Pradhan Mantri Gram Sadak Yojana (2) Swarna Jayanti Gram Swarozgar Yojana
(3) TRYSEM (4) Indira Aawas Yojana
(5) None of these
Q.19. During which of the following plan period annual growth rate of G.D.P. was registered as 3.6%?
(1) First five year plan (2) Second five year plan (3) Fourth five year plan
(4) Fifth five year plan (5) None of these
Q.20. Call money is related to
(1) Share Market (2) Government Securities (3) Post Office Deposit
(4) Banks Deposit (5) None of these
Q.21. Finance Commisson is included in of Indian Constitution.
(1) Article 340 (2) Article 280 (3) Article 360
(4) Article 390 (5) None of these
Q.22. Which of the following is/are TRUE about the "Village Gram Bank Scheme" ?
(1) The scheme is launched to provide safeguards against Starvation during the period of natural
calamities or lean season.
(2) The gram banks are set up in food scare areas like drought prone areas.
(3) Village Panchayats who were running Village Grain Banks earlier are now not authorized to
run.
(4) Both (1) and (2)
(5) None of these
GENERAL AWARENESS Mahendra's
Q.23. NAMA word is related to which organisation ?
(1) World Trade Organisation (2) UNCTAD (3) International Monetary Fund
(4) World Bank (5) None of these
Q.24. SEBI is related to -----------
(1) Bank (2) Government of India (3) Share Market
(4) State Government (5) None of these
Q.25. What is the main function of National Planning Council ?
(1) Suggestions to Planning Commission (2) Formation of Committee
(3) Formation of Five Year Plan (4) Formation of Socio-Economic Programme
(5) None of these
Q.26. Which of the following services is NOT provided by the post offices in India ?
(1) Savings Bank Scheme (2) Retailing of Mutual Funds
(3) Sale of Stamp Papers (4) Issuance of Demand Drafts
(5) None of these
Q.27. Banks are required to pay how much percentage of their net bank credit to Priority Sector in
advance ?
(1) 5% (2) 7% (3) 10% (4) 15% (5) None of these
Q.28. Economic Planning comes under which of the following lists ?
(1) Union List (2) Concurrent list (3) State List
(4) 1 and 3 (5) None of these
Q.29. Which of the following is/are correct about the 'Operational Risk' as used in the field of banking?
A. Risk of loss due to inadequate or failed internal process.
B. Risk of loss due to natural calamities.
C. Loss occurred due to non compliance of legal procedures.
(1) Only A (2) Only B (3) Only C (4) All (5) None of these
Q.30. The word "Code Sharing" is most used in which of the following industries ?
(1) Information Technology (2) Defense Industry (3) Nuclear Industry
(4) Air Transport Industry (5) None of these
Q.31. WADA (World Anti Doping Agency) is related with-----------.
(1) Insurance (2) Government Securities (3) Dope Testing
(4) Stock Exchange (5) None of these
Q.32. Bank Rate is-----------
(1) Rate at which commercial banks borrow long term funds from central Bank.
(2) Rate at which commercial banks borrow short term funds from central bank.
(3) Currently Bank rate is 8.5%
(4) Both 1 and 3
(5) None of these
Q.33. Which of the following statements is/are TRUE about Share Market ?
(A) Share Market is operated in stock exchange.
(B) In India Saurastra stock exchange and Kanpur stock exchanges are closed recently.
(C) In India 22 Stock Exchange are in operating condition.
(1) Only A (2) A and B (3) Only B (4) All (5) None of these
Q.34. Which of the following organisations releases "World Economic Outlook" Report ?
(1) Asian Development Bank (2) World Bank
(3) International Monetary Fund (4) World Trade Organisation
(5) None of these

Mahendra's GENERAL AWARENESS


Q.35. Money laundering is related -----------
(1) To reduce the number of families which lives below poverty line.
(2) To reduce the NPA of Banks.
(3) To reduce the Black Money.
(4) To reduce the unemployment rate.
(5) None of these
Q.36. In which year Public Debt Act was passed ?
(1) 1938 (2) 1940 (3) 1944 (4) 1948 (5) None of these
Q.37. Which of the following is not included in Indian Economy ?
(1) M1 (2) M
2 (3) M
3 (4) both M and M (5) None of these
1 3

Q.38. National Income indicates -


(1) The number of families below poverty line. (2) Goods and Services in Economy
(3) Growth of Economy (4) 2 and 3
(5) None of these
Q.39. The first five year plan was started in ---------.
(1) 1939 (2) 1949 (3) 1951 (4) 1956 (5) None of these
Q.40. What is the full form of ASBA ?
(1) Amount Saved and Balance in Account. (2) Account Savings & Bank Ascertainment.
(3) Applications Supported by Blocked Amount. (4) Already Savings Blocked Amount.
(5) None of these
Q.41. The debenture holders of a company are its_____.
(1) Employees (2) Debtors (3) Creditors (4) Partners (5) None of these
Q.42. Ministry of Communications and IT has launched a social security scheme the Service Discharge
Benefit Scheme (SDBS) -------------.
(1) To provide financial security to Gramin Dak Sevaks
(2) To provide insurance for the rural poor
(3) To provide risk cover for the urban slums
(4) To provide financial assistance for the gram panchayats
(5) None of these
Q.43. In which year National Bank for Agriculture and Rural Development (NABARD) was established in _____.
(1) 1991 (2) 1982 (3) 1952 (4) 1968 (5) None of these
Q.44. In banking terminology bad loans refer to ------------------.
(1) Bad debts (2) Non Performing assets
(3) Underwriting assets (4) Fictitious assets
(5) None of these
Q.45. Which of the following department is not a part of Finance Ministry of India ?
(1) Department of Revenue (2) Department of Expenditure
(3) Department of Economic Affairs (4) Department of Industrial Policy & Promotion
(5) None of these
Q.46. Which of the following is the name of a Private Sector Bank in India ?
(1) IDBI Bank (2) Axis Bank (3) Corporation Bank
(4) UCO Bank (5) All are Private Banks
Q.47. What is an Indian Depository Receipt ?
(1) A deposit account with a Public Sector Bank
(2) A depository account with any of Depositories in India
(3) An instrument in the form of depository receipt created by an Indian depository against
underlying equity shares of the issuing company
(4) An instrument in the form of deposit receipt issued by Indian depositories
(5) None of these
GENERAL AWARENESS Mahendra's
Q.48. Which is included in Capital budget ?
(1) Income received from public borrowings (2) Income received from tax-sources
(3) Income received from non-tax sources (4) All of the above
(5) None of these
Q.49. Mobile banking is set to get a boost from IMPS, which stands for----------------.
(1) Inter-Bank Mobile Payment Service (2) Inter-Bank Money Portability Service
(3) Intra-Bank Mobile Payment System (4) Inter-Bank Money Preventing System
(5) None of these
Q.50. What is the full form of NSDL?
(1) National Securities Depository Limited (2) National Securities Demanding Liability
(3) National Sample Depository Limited (4) National Sample Driven Land
(5) None of these
Q.51. Consider the following statements and state which of them are true ?
(A) Govt. launched service discharge benefit scheme for post retirement security of about
three lakh Gramin Dak Sevaks.
(B) Govt. will deposit two hundred rupees per month in accounts of Gramin Dak Sevaks working
in far flung areas.
(C) Scheme is being functionized by utilizing funds from PFRDA.
(D) Government will provide over 70 crore Rs. annually for this scheme.
(1) B and D (2) A, C and D (3) All are true
(4) A and C (5) None of these
Q.52. In 2011-12 Central Government launched a new Scheme 'Swabhimaan'. Which statements are true
about it ?
(1) It is a programme on Financial Inclusion
(2) Opening 5 crore new rural bank accounts.
(3) Providing branchless banking through technology.
(4) Covering all 73000 unbanked rural habitations with over 2000 population.
(5) All of the above
Q.53. Which of the following terms is/are related with finance and banking ?
(1) Velocity (2) Displacement (3) Badla
(4) Centigrade (5) None of these
Q.54. Tarapore Committee submitted its report on "Full Convertibility on Rupee" in-
(1) American Depository Receipt (2) Current Account (3) Capital Account
(4) 2 & 3 Both (5) None of these
Q.55. Scheme inaugurated helpline 'Udyami' is to assist----------.
(1) Rural Unemployed (2) Large Capital Industries
(3) Only female entrepreneus (4) Farmers introducing technology in farming
(5) Micro, Small and Medium enterprises
Q.56. Consider the following statements about newly launched Mahila Kisan Sashaktikaran Pariyojana
and State which of the following is/are the objectives of MKSP ?
(A) To create sustainable agricultural livelihood opportunities for women in agriculture
(B) To ensure food and nutrition security at the household and community level.
(C) To enhance the productive participation of women in agriculture.
(1) Only A (2) Only C (3) All are true
(4) A and B (5) None of these
Q.57. Which of the following is not Development Bank/Non Commercial Bank ?
(1) SIDBI (2) National Housing Bank (3) IDBI
(4) NABARD (5) None of these
Mahendra's GENERAL AWARENESS
Q.58. Which of the following is another name for Bank of Sweden Prize ?
(1) Noble Prize for Literature (2) Noble Prize for Peace
(3) Man Booker Prize (4) Noble Prize for Medicine
(5) None of these
Q.59. Market Cost of a product includes ____.
(1) Factor Cost (2) Advertisement Cost
(3) Transportation Cost (4) All of above
(5) None of these
Q.60. In the first phase of nationalisation of banks how many banks were nationalised ?
(1) 19 (2) 26 (3) 27 (4) 6 (5) None of these
Q.61. Which among the following will settle the grievances of customers of banks?
(1) State Bank of India (2) Banking Ombudsmen (3) Local Courts
(4) Nationalised Banks (5) None of these
Q.62. What was the targeted growth rate for 2012-17 in the 12th Five Year Plan?
(1) 8% (2) 10-11% (3) 9.6% (4) 9.5-9.75% (5) None of these
Q.63. Gilt - Edged market means-------.
(1) Bull Market (2) Gun Market
(3) Market of Government Securities (4) Market of Pure Metal
(5) None of these
Q.64. Many times we have read about ULIP in newspapers. ULIP stands for ________.
(1) United Loaded Investment Plan (2) Universal Loan Investment Plan
(3) Unit Life Insurance Plan (4) Unit Linked Insurance Plan
(5) None of these
Q.65. Devaluation of currency results in ____
(1) Increased export and import
(2) Trade deficit
(3) Increased export and improvement in balance of payment
(4) Increased import & foreign reserve deficiency
(5) None of these
Q.66. The Govt. of India replaced the FERA by _______.
(1) FEMA (2) FRBMA (3) Monopolies Act
(4) FRTP (5) None of these
Q.67. Banking which involves mobilizing large part of the deposits in risk free assets such as government
securities is called _______.
(1) Overseas Banking (2) Offshore Banking (3) Islamic Banking
(4) Narrow Banking (5) None of these
Q.68. What is the full form of REER?
(1) Real Earning Emergent Ratio (2) Real Elective Effective Ratio
(3) Real Effective Exchange Rate (4) Role Effective Earned Rate
(5) None of these
Q.69. Consider the following statements and state which of them are true?
A. The planning commission's 40 members steering committee on Health will advise panel on
the twelfth five year plan.
B. The 40 member committee on Health is chaired by Syeda Hameed.
C. The steering committee will review the National Health Policy, as well as explore the possibility
of adopting the Right to Health as an approach with focus on women, children & preventive &
curative healthcare.
(1) Only B (2) Only C (3) A & C (4) All are true (5) None of these

GENERAL AWARENESS Mahendra's


Q.70. IRDA announced a Health Insurance Policy Portability. This will be implemented from _______.
(1) 1 April, 2011 (2) 1 August, 2011 (3) 1 September, 2011
(4) 1 July, 2011 (5) None of these
Q.71. Which of the following term is used in banking and finance ?
(1) Cathode (2) Diabetic (3) Equity (4) Obese (5) None of these
Q.72. Consider the following & state which of them are TRUE ?
(A) New Foreign Contribution Regulation Act has come into force.
(B) Under the law, no political party can receive foreign funds as donation.
(C) The act says in case of any organisation receiving funds over 10 lakh, bank will immediately
inform govt. to enable agencies to track funds.
(D) There are over 40000 organisations recieving foreign contribution in country out of which only
18000 report the inflow of funds & submit their accounts.
(1) All are true (2) Only D (3) B and D (4) Only C (5) None of these
Q.73. Committee related with the merger of regional rural banks with their sponsor banks is-------.
(1) Khusro Committee (2) Khan Committee
(3) Malegam Committee (4) Hilton Young Committee
(5) None of these
Q.74. Functions of NBFC includes-----------.
(1) Equipment Leasing (2) Term Deposits
(3) Housing Finance & Investment in Financial Securities (4) 1 & 3
(5) None of these
Q.75. What does PMLA stands for ?
(1) Primary Money Launching Authority (2) Preventing Money Launching Act
(3) Promoting Money Laundering Act (4) Prevention of Money Laundering Act
(5) None of these
Q.76. Bad debts recovered is a ------for the organisation.
(1) Expense (2) Debt (3) Income (4) Asset (5) None of these
Q.77. Which of the following term is not used in Banking/Finance ?
(1) Revenue (2) Epicenter (3) Commercial Papers
(4) Consolidated Fund (5) None of these
Q.78. Which of the following maintains the RIDF (Rural Infrastructure Development Fund) ?
(1) RBI (2) IDBI (3) SIDBI (4) NABARD (5) None of these
Q.79. What does a rise in SENSEX means ?
(1) Rise in prices of all companies belonging to U.K.
(2) Overall rise in prices of shares of companies registered in BSE.
(3) Overall rise in prices of shares of companies registered in NSE.
(4) Rise in prices of shares of companies of America.
(5) None of these
Q.80. Which of the following in India formulates the fiscal policy?
(1) Finance Commission (2) RBI (3) Ministry of Finance
(4) Planning Commission (5) None of these
Q.81. Indian company having FDI will be permitted to make downstream investment in LLPs. What is the
full form of LLP ?
(1) Limited Language Performance (2) Limited Liability Partnership
(3) Linguistic Liability Participation (4) Limited Lasting Participation
(5) None of these

Mahendra's GENERAL AWARENESS


Q.82. Which one of the following launched social security scheme for 'Gramin Dak Sevaks' ?
(1) Ministry of Economic Affairs (2) Ministry of Rural Infrastructure
(3) Ministry of Communications & IT (4) Ministry of Rural Development
(5) None of these
Q.83. What is the Platinum Jubilee Year of the RBI ?
(1) 2007-2008 (2) 2008-2009 (3) 2012-2013 (4) 2014-2015 (5) 2009-2010
Q.84. National Food for Work Programme was launched in--------------.
(1) 2004 (2) 2010 (3) 2001 (4) 2008 (5) None of these
Q.85. Kisan Credit Card scheme was launched in the year________.
(1) 2000 (2) 1998 (3) 1996 (4) 1992 (5) None of these
Q.86. Credit which is available at low rates of Interest is defined as _______.
(1) Hot money (2) Fiat money (3) Token money (4) Cheap money (5) None of these
Q.87. Which Bank is the apex bank of USA ?
(1) Citi Bank (2) Bank of America (3) Federal Reserve
(4) Central Bank of USA (5) None of these
Q.88. Which of the following statements are correct about Finance Commission ?
(A) It is a statutory body.
(B) It is constituted under article 280 of Indian constitution.
(C) Mr. K. C. Pant is the Chairman of 13 Finance Commission.
th

(1) Only A (2) B and C (3) A and B (4) A, B & C (5) None of these
Q.89. In Regional Rural Banks, What is the contribution of the Sponsor Banks ?
(1) 50% (2) 15% (3) 35% (4) 25% (5) None of these
Q.90. Which one of the following is not a part of the World Bank Group ?
(1) The International Finance Corporation (2) Asian Development Bank
(3) The Multi-lateral Investment Guarantee Agency (4) International Development Association.
(5) None of these
Q.91. The nature of convertibility of the Indian Economy is ______.
(1) Convertibility at capital account (2) Convertibility at trade account
(3) Free Convertible (4) Convertibility at current account
(5) None of these
Q.92. What is the full form of LERMS ?
(1) Liberalised Exchange Rate Management System
(2) Long Elimination Reserve Management System
(3) Liberalised Extended Related Management Stock
(4) Linguistic Exchange Reserve Management Scenario
(5) None of these
Q.93. Which of the following is the basic objective of R.B.I's monetary policy ?
(1) To control deficit in the balance of payment (2) To control public expenditure
(3) To control deficit of the budget (4) To control cost & availability of money
(5) None of these
Q.94. Stagflation refers to ________.
(1) High inflation in periods of full employment
(2) High deflation in periods of high unemployment
(3) Deflation in periods of stagnant employment
(4) High inflation in periods of high unemployment
(5) None of these

GENERAL AWARENESS Mahendra's


Q.95. Which of the follwoing measures is not taken by government to control inflation ?
(1) It increases the rate of taxes (2) It increases cash reserve ratio
(3) It increases the imports (4) It increases the public debt
(5) None of these
Q.96. The object of introducing base rate by RBI is_____.
(A) Enhancing transparency in lending rates of banks
(B) Enabling better assessment of transmission of monetary policy
(C) Keeping interest rates low
(1) A and B (2) Only C (3) Only B (4) All of above (5) None of these
Q.97. Which of the following defines Commercial Papers?
(1) Long term debt issued by private corporations.
(2) Short-term and unsecured promissory notes issued by corporations with-very high credit standings.
(3) Savings instrument, in which funds must remain on deposit for a specified period, and
premature withdrawals incur interest penalties.
(4) 1 and 3
(5) None of these
Q.98. Which of the following bill is regarding timely delivery of information & documents like birth & death
certificates & revenue records to people & empowering them to file complaints against delays?
(1) Jan Lokpal Bill (2) Janhit Guarantee Bill
(3) Right to Education (4) Jansewa Guarantee Bill
(5) None of these
Q.99. Which of the following terms is not related with banking/finance?
(1) Call Option (2) Coupon (3) Stock Splits (4) Ablation (5) None of these
Q.100. What is the maturity period of Treasury Bills issued by Government of India?
(1) 14 days and 84 days (2) 182 days and 364 days
(3) 18 days and 36 days (4) 91, 182 & 364 days
(5) None of these
Q.101. Many a times Government raises additional resources through taxation & allocation of expenditure
as per its requirements. It is known as ---------
(1) Credit Policy (2) Fiscal Policy (3) Monetary policy
(4) Taxation Policy (5) None of these
Q.102. In which year did second phase of nationalization of banks took place?
(1) 1969 (2) 1979 (3) 1980 (4) 1990 (5) None of these
Q.103. SLR is maintained by banks on which of the following
(1) Demand and Time Deposits (2) Net Demand and Time Deposits
(3) Demand and Time Liabilities (4) Net Demand and Time Liabilities
(5) None of these
Q.104. What is the base year for new series of IIP (Index of Industrial Production) ?
(1) 1993-94 (2) 1994-95 (3) 2003-2004 (4) 2002-2003 (5) 2004-2005
Q.105. Which of the following is not an objective of fiscal policy of Government of India ?
(1) Price Stability (2) Full Employment
(3) Equal distribution of Income and Wealth (4) Regulation of Inter State Trade
(5) None of these
Q.106. Increase in Fiscal Deficit leads to ______.
(1) No change in BOP (2) Increase in Balance of Payments
(3) Reduction in Balance of Payments (4) 1 and 2
(5) None of these
Mahendra's GENERAL AWARENESS
Q.107. 'Growth with Stability' was the aim of which of the following Five Year Plans ?
(1) Second Plan (2) Third Plan (3) Fourth Plan (4) Fifth Plan (5) None of these
Q.108. Which of the following is not a part of India's Money Market ?
(1) Money Lenders (2) Indian Gold Council (3) Bill Markets
(4) Banks (5) None of these
Q.109. When more than one banks are allowing credit facilities to one party in coordination with each other
under a formal arrangement, the arrangement is called __.
(1) Securitization (2) Participation (3) Consortium (4) Consolidation (5) None of these
Q.110. International Bank for Reconstruction and Development is related to _____.
(1) World Bank (2) ADB (3) IMF (4) Federal Bank (5) None of these
Q.111. A banking system in which a big bank has a number of branches in different parts of the country is
known as ___.
(1) Unit Banking (2) Branch Banking (3) Mixed Banking
(4) Chain Banking (5) None of these
Q.112. Which of the following is an example of near money ?
(1) Fiat money (2) Bank Draft (3) Treasury Bills
(4) 50 paise coins (5) None of these
Q.113. Raising or lowering of the central bank discount rate is known as ____.
(1) Bank rate policy (2) Open market operation
(3) Cash reserve ratio (4) 2 and 3
(5) None of these
Q.114. 'Gold Tranche' refers to ___.
(1) a credit system granted by IMF to its members. (2) A loan system of the World Bank
(3) A credit system granted by WTO to its members. (4) One of the operations of a Central Bank
(5) None of these
Q.115. National Food Security Mission was launched in the year ___.
(1) 2005-06 (2) 2006-07 (3) 2007-08 (4) 2008-09 (5) None of these
Q.116. Which is a legal tender in a modern economy amongst the following ?
(1) Currency notes (2) Cheques (3) Bank Draft
(4) Promissory notes (5) None of these
Q.117. Which of the following doesn't relates with banking or finance ?
(1) Devaluation (2) Hundis (3) Flare (4) Gearing (5) None of these
Q.118. ESOP means -
(1) Employee stock option play (2) Employee shair object plan
(3) Employee stock option plan (4) Enter shair object plan
(5) All of these
Q.119. Which of the following is not a characteristics of Fiat money?
(1) Fiat money is created and issued by the state
(2) Fiat money is inconvertible by law
(3) Its value as a commodity is as great as its value as money
(4) It is a legal tender
(5) None of these
Q.120. A banking system where all operations are confined to a single office is known as _____.
(1) Unit Banking (2) Group Banking (3) Chain Banking
(4) Branch Banking (5) None of these
Q.121. Which of the following is not a Financial/Economic term?
(1) Marginal Utility (2) Dumping (3) Pipette
(4) Break Even Point (5) None of these
GENERAL AWARENESS Mahendra's
Q.122. What is the full form of FIMMDA?
(1) Fined Income More Money Derivatives Authority
(2) Fixed Income Measuring Money Dumping Association
(3) Fixed Income Money Market and Derivatives Association
(4) Fixed Initial Money Market Development Authority
(5) None of these
Q.123. An organisation which pools money from investors & invests in stock, bonds, shares is called ______.
(1) Bancassurance (2) A Bank (3) Insurance company
(4) Mutual Fund (5) None of these
Q.124. Which of the following term in related with Economy?
(1) Absolute Zero (2) Amplitude (3) Soft patch (4) Buoyancy (5) None of these
Q.125. Which of amongst the following is not the aim of Debt Management?
(1) Stabilizing prices (2) Social service fund
(3) Proper timing & issuing of government bonds (4) Cost effective services
(5) None of these
Q.126. When RBI wants to reduce liquidity in the banking system-
(A) It increases the CRR
(B) It increases the SLR
(C) It increases the Repo Rate
(1) Only A (2) Only B (3) Only C (4) All A, B and C (5) None of these
Q.127. Negotiated Dealing System Relates to _____.
(A) Trading at stock markets
(B) Settlement of secuirty dealings
(C) Trading in government securities
(1) Only A (2) Only B (3) Only C (4) All A, B and C (5) None of these
Q.128. The Base rate of each bank is to be reviewed after every_______.
(1) Quarter (2) Month (3) Year (4) Week (5) Day
Q.129. Gross Domestic Product is a measure of_______.
(1) a country's internal trade relation (2) a country's domestic economic activities
(3) a country's financial position (4) a country's foreign trade relation
(5) None of these
Q.130. In which of the following years, the concept of Micro Finance launched ?
(1) 1982 (2) 1992 (3) 2002 (4) 1972 (5) None of these
Q.131. The network for common data communication for Banks and Financial Institutions launched by RBI
is________
(1) INTERNET (2) INFINET (3) SWIFT (4) BANKNET (5) None of these
Q.132. If a loan is repaid by a cheque, ____.
(1) Total bank reserves in the system decreases. (2) Total bank reserves are not affected.
(3) Increase in total bank reserves in the system. (4) 1 and 3 both
(5) None of these
Q.133. Consider the following statements about Gold Standard and state which of them are true ?
(A) Gold Standard means there exists a fixed relationship between any two currencies called
the mint parity.
(B) It also means the exchange rate is determined by demand and supply between the gold points.
(C) Each nation defines the price of gold in terms of its currency and then stands ready to buy
and sell.
(1) Only C (2) Only B (3) All are true (4) Only A (5) None of these
Mahendra's GENERAL AWARENESS
Q.134. Which of the following is not a component of bank deposits ?
(1) Fixed deposits (2) Current A/C deposits (3) Recurring deposits
(4) Safe deposits (5) None of these
Q.135. The new issue market is connected with _______.
(1) New issue of treasury bills (2) New issue of currency notes
(3) Issue of IPO (4) All of above
(5) None of these
Q.136. Which of the following is a new instruments permitted by RBI to bank under Tier II for Capital
adequacy purposes and what is its minimum maturity period ?
(1) Innovative perpetual debt instrument, 10 years
(2) Non-redeemable Cumulative Capital Instrument, 10 years
(3) Perpetual preference shares, 15 years
(4) Redeemable preference shares, 15 years.
(5) None of these
Q.137. Which of the following interest rates are not fixed by RBI and left to the discretion of the Bank
concerned ?
(A) Saving Bank deposits (B) DRI Advances
(C) Ceilling rates on FCNR/NRE deposit accounts (D) Ceiling rates on export credit facilities
(1) A and D (2) A and C (3) Only A (4) Only D (5) None of these
Q.138. Which of the following liabilities is not taken as part of Time and demand liabilities for the purpose of
CRR ?
(1) Fixed deposits (2) Saving bank account (3) Recurring deposits
(4) Paid-up capital (5) None of these
Q.139. Credit creation capacity of commercial banks is determined by :
(A) Cash withdrawls from the banking system by the public
(B) Cash reserve ratio to be maintained by Commercial banks
(C) Supply of currency notes by the central bank
(D) Willingness of the borrowers to take loans from commercial banks.
(1) A, C and D (2) B, C and D (3) A, B and D (4) A, B and C (5) None of these
Q.140. Money supply in India can be increased if_____
(A) RBI puts more paper money for circulation
(B) The commercial banks expand their credit operations
(C) The Central Government gives more grants to the states
(D) The Government of India borrows from the RBI
(1) A, B and C (2) B, C and D (3) A, C and D (4) A, B and D (5) None of these
Q.141. What is meant by a Scheduled Bank ?
(1) A Bank functioning under provisions of Banking Regulation Act. 1949
(2) A Bank included in 2nd schedule of RBI Act 1934
(3) A Bank incorporated under Banking Companies Act. 1956
(4) A Bank authorised to do Banking functions
(5) None of these
Q.142. Which one of the following is not a Quantitative credit control measure of the Reserve Bank of India ?
(1) Bank rate (2) Statutory cash reserve requirement
(3) Statutory liquidity ratio (4) Moral Suasion
(5) None of these
Q.143. Fiscal Deficit concept was introduced in Indian economy by
(1) Amartya sen (2) V. K. R. V. Rao (3) S. Chakravarty
(4) Madhu Dandavate (5) None of these
GENERAL AWARENESS Mahendra's
Q.144. Consider the following statements regarding Special Drawing Rights (SDR)-
(A) SDR is known as the paper gold
(B) It is an international unit of account
(C) The IMF pays no interest on all holdings of SDRs, kept in the special drawing account
(D) The Quotas of all currencies in the IMF General account are also valued in terms of the
SDR
(1) A, B and C (2) B, C and D (3) A, B and D (4) A, C and D (5) None of these
Q.145. Consider the following statement and state which among those is/are correct ?
Regional rural banks in India___
(A) Are required to assist the weaker sections
(B) Are to help small and marginal farmers only
(C) Have free access to liberal financial assistance from NABARD
(D) Have a limited area of operation
(1) A, C and D (2) Only B (3) B and C (4) A and D (5) None of these
Q.146. In deficit financing, the government borrows money from _____.
(1) RBI (2) Public (3) IMF (4) Capitalists (5) None of these
Q.147. CSO was established in the year ______.
(1) 1952 (2) 1957 (3) 1954 (4) 1951 (5) None of these
Q.148. On the recommendation of which committee Rail Budget was seperated from General Budget in
1924 ?
(1) Acworth Committee (2) Hilton Young Committee
(3) Patnayak Economy Committee (4) Brettenwood Committee
(5) None of these
Q.149. Treasury bills also called as T- bills are the part of ____.
(1) Unorganized Market (2) Capital Market (3) Money Market
(4) Commodities Market (5) None of these
Q.150. Planned expenditure in India is made from _______.
(1) Internal debt and other resources (2) Assistance from IMF
(3) Assistance from ADB (4) 1, 2 and 3
(5) None of these
Q.151. Primary Deficit is obtained by subtracting ____ from fiscal deficit.
(1) Revenue Deficit (2) Depreciation (3) Borrowings
(4) Interest Payment (5) None of these
Q.152. In India nationalization of banks took place in how many phases ?
(1) 3 (2) 2 (3) 4 (4) 1 (5) None of these
Q.153. Fixed deposits are for the bank______
(1) Fixed Assets (2) Time liability (3) Demand liability
(4) Current Assets (5) None of these
Q.154. Which of the following come under non-plan expenditure ?
(A) Subsidies.
(B) Interests payments
(C) Defence Expenditure.
(D) Maintenance expenditure for the infrastructure created in previous plans.
(1) A and B (2) A and C (3) B and D (4) A B and D (5) None of these
Q.155. Which of the following does not come under tertiary sector of the economy ?
(A) Trade (B) Industrial Manufacturing (C) Industrial Construction
(1) Only B (2) Only C (3) B and C (4) Only A (5) None of these
Mahendra's GENERAL AWARENESS
Q.156. Which of the following is not a direct tax ?
(1) Estate duty (2) Sales Tax (3) Wealth Tax (4) Corporation Tax (5) None of these
Q.157. Which of the following schedules of Indian constitution includes Banking ?
(1) 8th (2) 7
th
(3) 5
th
(4) 6
th
(5) None of these
Q.158. 'Open Market operation' is a part of ______
(1) Labour Policy (2) Fiscal Policy (3) Monetary and Credit Policy
(4) Income Policy (5) None of these
Q.159. What is a soft currency ?
(1) A currency whose exchange rate has a tendency to fluctuate
(2) A currency which can be easily used for settling any International transactions.
(3) A currency which can not be used for settling international transactions because it does
not command a value in the international market
(4) A currency which is available by way of soft loans from international agencies
(5) None of these
Q.160. Which of the following is not a Banking/ Economy Term ?
(1) Factoring (2) Blue Chip (3) Insider Trading
(4) Capital gain (5) Digital Signature
Q.161. Which one of the following sources is not considered as a source of Government revenue ?
(1) Taxes (2) Surplus of Public Enterprises
(3) Mobilisation of internal loan and deposits (4) Transfer Payments
(5) None of these
Q.162. As we know that Reserve Bank of India changes the CRR. Which of the following statement is
correct in this regard ?
(A) Reduction in CRR increases the liquidity position within Indian banks.
(B) Decrease in CRR decreases lending capacity of banks.
(C) Increase in CRR does not affect the liquidity position.
(1) Only A (2) A and C (3) Only C (4) Only B (5) None of these
Q.163. Discount and Finance House of India Limited was set up by RBI in_______
(1) 1988 (2) 1989 (3) 1987 (4) 1990 (5) None of these
Q.164. The relationship between a banker and a customer is _______
(1) That of a debtor and a creditor (2) Primarily that of a debtor and a creditor
(3) That of a creditor and a debtor (4) Both 1 and 2
(5) None of these
Q.165. FCNR accounts can be opened and maintained as _________
(1) Savings Bank Account (2) Term Deposit Accounts
(3) Recurring Deposits (4) Current Accounts
(5) None of these
Q.166. Current Account Deficit on the Balance of payment reflects_________
(1) Deficit on Trade Account .
(2) Deficit on Invisibles Account.
(3) Deficit on Trade and Invisibles Account.
(4) Deficit onTrade Invisibles and Capital Account.
(5) None of these
Q.167. Which one of the following is an example of joint supply ?
(1) Sugar and Tea (2) Diesel and Bus (3) Ink and Fountain Pen
(4) Wool and Mutton (5) None of these
GENERAL AWARENESS Mahendra's
Q.168. Consider the following statements regarding loan syndication and state which among those are
true?
(A) More than one debtors applying for a loan to one creditor.
(B) More than one creditors coming together to provide single loan.
(C) More than one creditors coming toghter to provide multiple loans.
(1) A and B (2) only C (3) only B (4) A and C (5) None of these
Q.169. A 'closed economy is an economy in which
(1) The deficit financing takes place
(2) Only exports takes place
(3) Neither exports nor imports takes place
(4) The money supply is full
(5) None of these
Q.170. If RBI adopts an expansionist open market operations policy, this means that it will.
(1) offer commercial banks more credit in open market
(2) sell securities in the open market
(3) openly announce to the market that it intends to expand credit.
(4) buy securities from non- government holders
(5) None of these
Q.171. Moral Suasion is the instrument of
(1) Trade Policy (2) Fiscal Policy (3) Budget
(4) Monetary Policy (5) None of these
Q.172. If an Indian Citizen renting a flat in London it is to be included in .
(1) GDP of India (2) GNP of London (3) GNP of India
(4) GDP of London (5) None of these
Q.173. Parallel economy emerges due to ..
(1) Tax Avoidance (2) Tax Evasion (3) Tax Compliance
(4) Tax Estimation (5) None of these
Q.174. The type of note issue system followed in India is
(1) Proportional Fiduciary System (2) Fixed Fiduciary System
(3) Minimum Reserve System (4) Minimum Fiduciary System
(5) None of these
Q.175. Government securities are considered liquid because they are .
(1) Backed by the government treasury .
(2) Convertible into the other types of saving deposits
(3) Stable in value
(4) Quickly & easily Marketable
(5) None of these
Q.176. Who formulates the Monetary policy in India ?
(1) Planning commission (2) Finance ministry (3) RBI
(4) SEBI (5) None of these
Q.177. The Tax on import and export is known as____.
(1) Income Tax (2) Trade Tax (3) Custom Duty
(4) Excise Duty (5) None of these
Q.178. The Integrated Action plan recently approved by the Government of India is related with ____.
(1) Natural disaster (2) Inflation (3) Terrorism
(4) Naxalism (5) None of these
Mahendra's GENERAL AWARENESS
Q.179. Which of the following sets of sources of revenue belongs to the Union Government alone ?
(1) Gift Tax, Holding Tax (2) Sales Tax, Income Tax
(3) Customs Duties, Corporation Tax (4) Wealth Tax, Land Revenue
(5) None of these
Q.180. The Government of India passed the "Recovery of Debts Due to Banks and Financial Institutions
Act" in ______.
(1) 1993 (2) 1992 (3) 1994 (4) 1990 (5) None of these
Q.181. Who among the following acts as agent of Government of India in respect of her membership of
IMF?
(1) Ministry of Finance (2) Reserve Bank of India (3) SEBI
(4) NABARD (5) None of these
Q.182. Index "Residex" is associated with______.
(1) Share Prices (2) Mutual Fund Prices (3) Prices Inflation Index
(4) Land Prices (5) None of these
Q.183. The 'Dolex' Indicator is related to which of the following Financial Markets ?
(1) New York (2) Mumbai (3) Tokyo (4) Hong Kong (5) None of these
Q.184. Industrial exit policy means _____.
(1) Forcing foreign companies to leave India.
(2) Forcing business units to move out of congested localities.
(3) Allowing manufactures to shift their line products.
(4) Allowing business units to close down.
(5) None of these
Q.185. All commercial banks demand deposit liabilities may decrease as a result of _____.
(1) An inflow of cash (2) A decrease in loans
(3) An increase in security holdings (4) The chasing of a cheque by an individual or a firm.
(5) None of these
Q.186. Which of the following is not a component of foreign aid of a country ?
(1) Bilateral grant (2) Multilateral grant
(3) Loans from international financial institutions (4) Commercial borrowing
(5) None of these
Q.187. Which of the following represents priority sector lending by commercial banks in India ?
(A) Lending to heavy industries.
(B) Lending to foreign companies.
(C) Lending to agriculture, small scale industries and the weeker sections of the society.
(1) A and B (2) Only C (3) A, B and C (4) A and C (5) None of these
Q.188. In the post-liberalization period, the share of direct taxes in gross tax revenue in India has
________.
(1) Increased (2) Decreased
(3) First increased and then decreased (4) Remained the same
(5) None of these
Q.189. If the government chooses to spend money freshly printed by the RBI against government
securities,it should be kept in which among the following?
(1) Fiscal adjustment (2) Retrenchment (3) Mandatory Spending
(4) Deficit Financing (5) None of these

GENERAL AWARENESS Mahendra's


Q.190. Which of the following defines letter of credit ?
(1) A contractual agreement made between different parties to compensate for any damages or losses.
(2) It is a letter from a bank, guaranteeing that a buyers payment to a seller will be received
on time and for the correct amount.
(3) A solemn promise or agreement to do or refrain from doing something.
(4) A security interest on real property grant to a lender.
(5) None of these
Q.191. Which of the following term is related to Banking/ Finance ?
(1) Acne (2) Ativan (3) Empire (4) Paradox (5) Encryption
Q.192. The Global Hunger Index is compiled by the ................
(1) Food and Agricultural Organisation
(2) United Nations Development Programme
(3) International Food Policy Research Institute
(4) United Nations World Food Programme
(5) None of these
Q.193. Which of the following is shown on Liabilities side of a banks balance sheet ?
(1) Reserve fund (2) Loans & advances (3) Bills discounted
(4) Bills for collection (5) None of these
Q.194. The commercial bank do not have unlimited power of credit creation. Their power to create credit is
limited due to -
(1) Amount of cash received by the bank (2) Banking habits of the people
(3) Leakages in credit creation (4) 1 and 3
(5) None of these
Q.195. A type of cheque which is more than six months old is a -
(1) Mutilated cheque (2) Crossed cheque (3) Stale cheque
(4) Post-dated cheque (5) None of these
Q.196. Cash Deposit Ratio means ____
(1) The percentage of total cash money received as deposits by banks
(2) The percentage of cash in hand balance with central bank to the aggregate deposits
(3) The percentage of cash in hand with amount of loan given to customer
(4) The percentage of assets to liabilities of bank
(5) None of these
Q.197. How many food parks do the government aims to set up across the country in 12 plan ?
th

(1) 50 (2) 30 (3) 20 (4) 40 (5) None of these


Q.198. The main function of Central Statistical Organisation is ____
(1) To determine the money supply (2) To determine prices
(3) To publish national income estimates (4) To collect employment details
(5) None of these
Q.199. Which amongst the following promoted the concept of self help groups (SHGs) for financing the poor ?
(1) RBI (2) Union Ministry of labour
(3) NABARD (4) Union Ministry of Rural development
(5) None of these
Q.200. Human Capital refers to _____
(A) The attributes gained by worker through education and experience
(B) Skill, capacity and ability possessed by the population
(C) Material capital possessed by the human beings
(D) Population of the country
(1) A and B (2) B and C (3) A and C (4) B and D (5) All are true
Mahendra's GENERAL AWARENESS
Q.201. Legal Tender Money is____
(1) Accepted only by the Government
(2) Accepted by the people and the Government as per the law
(3) Not accepted for business purposes by the law
(4) Not accepted by the government
(5) None of these
Q.202. Stock exchange securities do not include ______
(1) Debentures of companies (2) Small debentures issued by port trusts
(3) Government promissory note (4) Participation certificates
(5) None of these
Q.203. Which of the following is not commercial bank ?
(1) Foreign Banks (2) Private Banks (3) RBI
(4) Regional Rural Banks (5) None of these
Q.204. A non - negotiable crossing is a _____
(1) Restricted crossing (2) Special crossing (3) Non- transferable crossing
(4) Parallel crossing (5) None of these
Q.205. Which of the following statements are true about Working Capital ?
(A) Working Capital is a financial metric which represents operating liquidity available to a
business, organization, or other entity, including government entity.
(B) Net working capital is calculated as current assets minus current liabilities
(C) The management of Working Capital involves managing inventories, accounts receivable
and payable, and cash.
(1) Only A (2) Only B (3) Only C (4) All are true (5) None of these
Q.206. Floating Exchange Rate is also referred to as________
(1) Real Exchange Rate (2) Controlled Exchange Rate
(3) Fixed Exchange Rate (4) Flexible Exchange Rate
(5) None of these
Q.207. Which of the following Negotiable Instruments can be crossed to the banks ?
(1) Cheques (2) Bills of Exchange (3) Drafts
(4) All of above (5) None of these
Q.208. Certificates of Deposits can be issued for a minimum period of______
(1) 1 year (2) 6 months (3) 4 months (4) 3 months (5) 45 days
Q.209. When does the central Bank increase the minimum statutory cash reserve ratio of the commercial
banks ?
(1) When the economy is in deflationary condition
(2) When the central Bank aims credit expansion
(3) When the economy is in recession
(4) 1, 2 and 3
(5) None of these
Q.210. Which one of the following represents capital adequacy ratio for commercial banks ?
(1) Ratio of capital to risk-weighted assets (2) Ratio of capital to advances
(3) Ratio of capital to short-term deposits (4) Ratio of capital to non-performing assets
(5) None of these

GENERAL AWARENESS Mahendra's


Q.211. Near Money is defined as an________
(1) Asset which is not usable as a medium of exchange but has as store of value.
(2) Asset that serves as a temporary medium of exchange and has a store of value.
(3) Asset which adequately fulfill Store of value function and is readily convertible into a medium
of exchange but is not itself a medium of exchange.
(4) Asset which has a store of value & 100% liquidity
(5) None of these
Q.212. Foreign Exchange Rates in India are determined by______
(1) RBI (2) SEBI (3) Planning Commission
(4) Market Forces of demand/supply (5) None of these
Q.213. Which of the following term is used in banking ?
(1) Listing (2) Open Market Operations
(3) Settlement Day (4) Stock
(5) None of these
Q.214. Which amongst the following banks used Tiny Cards with biometric identification as a part of the
financial inclusion ?
(1) SBI (2) PNB (3) ICICI (4) Axis (5) HDFC
Q.215. Treasury bills are sold in India by-
(1) SEBI (2) State Government (3) RBI
(4) Commercial Banks (5) None of these
Q.216. Which of the following assets are considered as near money ?
(1) Bond (2) Equity Shares (3) Time deposits
(4) Traveller's cheques (5) All of above
Q.217. Forced saving refers to____
(1) Taxes on individual income & wealth
(2) Compulsory deposits imposed on income tax payers
(3) Provident fund contributions of private sector employees
(4) Reduction of consumption consequent to a rise in prices
(5) None of these
Q.218. Which of the following would reduce the credit creation capacity of a commercial bank ?
(1) Time & Demand deposits (2) Loans (3) Cash in hand
(4) 1 & 2 (5) All of 1, 2 and 3
Q.219. Which of the following does not form a part of the foreign exchange reserves of India ?
(1) Foreign currency & securities held by the banks & corporate bodies
(2) Gold
(3) SDR
(4) Foreign currency assets
(5) None of these
Q.220. Gains in trade results from__
(1) Exporting as much as possible and receiving gold.
(2) Reallocation of existing goods between the two countries.
(3) The fact that exchange brings both specialization & reallocation of greater output & the
increased welfare in each country
(4) One country receiving both imports & exports
(5) None of these

Mahendra's GENERAL AWARENESS


Q.221. The Reserve Bank of India has been intervening in foreign exchange market of India and buying
dollors to ___.
(A) Check enhanced liquidity in the monetary system
(B) Build - up foreign exchange reserves for debt servicing and debt repayment.
(C) Prevent the rupee strenthening further there by acting as disincentive to export growth.
(1) Only A (2) A and B (3) Only C
(4) A and C (5) None of these
Q.222. Non-Performing assets of commercial banks means their loans _____.
(1) Fetching very low rate of Interests.
(2) For which interest/instalment has remained unpaid after due date.
(3) Given to sick industrial units .
(4) Have not been disbursed at all .
(5) None of these
Q.223. Which of the following is not the feature of Real Time Gross Settlement (RTGS) ?
(1) Payments are settled transaction by transaction
(2) Settlement of funds is revocable.
(3) Settlement is done in real time.
(4) It is a fully secured system which uses digital signatures and public key encryption for
safe and secure message transmission.
(5) None of these
Q.224. The definition of 'Banking' is given in _____.
(1) Negotiable Instrument Act. 1881. (2) RBI Act, 1934
(3) The Banking Regulation Act, 1949 (4) The Indian Contract Act, 1872
(5) None of these
Q.225. In the case of FCNR Accounts the payment of Interest is effected in _____.
(1) Indian Rupee (2) Only in pound
(3) Same currency in which deposit stands. (4) Yen
(5) None of these
Q.226. The commercial paper is issued to raise deposits by _____.
(1) Commercial Banks (2) Reserve Bank of India
(3) Every Non-Banking company (4) State Bank of India
(5) None of these
Q.227. The Methods of credit control used by the RBI can be divided into _______.
(1) Long-term and short term (2) Rural and Urban
(3) Qualitative and Quantitative (4) Monetary and Fiscal
(5) None of these
Q.228. Increase in Bank rate is generally followed by ____.
(1) An increase in market rates of interest
(2) A fall in market rates of interest
(3) A rise only in the deposit rates but not the lending rates.
(4) A rise only in the base rates
(5) None of these
Q.229. Which one of the following banks creates credit ?
(1) Reserve Bank of India (2) Commercial Bank
(3) Industrial Banks (4) Land Mortgage Banks
(5) None of these

GENERAL AWARENESS Mahendra's


Q.230. Which of the following is the least liquid asset ?
(1) Money (2) Shares (3) Machinery
(4) Special deposits (5) None of these
Q.231. Which of the following is /are the most important component of the liabilities of Commercial Banks in
India ?
(A) Demand deposits
(B) Time Deposits
(C) Inter-Bank liabilities
(1) Only A (2) Only B (3) A and C
(4) All A, B and C (5) None of these
Q.232. An increase in foreign exchange assets of Reserve Bank of India will mean_____.
(A) Fall in M
1

(B) Increase in interest rate


(C) Increase in high powered money
(1) Only C (2) A and C (3) Only B (4) Only A (5) None of these
Q.233. Which one of the following is not a method of credit control ?
(1) Bank Rate (2) Credit Deposit Ratio (3) Cash Reserve Ratio
(4) Statutory Liquidity Ratio (5) None of these
Q.234. The largest Nationalized Commercial Bank in India at present is ____.
(1) Reserve Bank of India (2) State Bank of India
(3) Central Bank of India (4) Bank of India
(5) None of these
Q.235. Reserve money includes _____.
(1) Time deposit with the Banks only (2) Currency with the public and cash with Banks
(3) Cash with Banks only (4) Cash with Banks and post office savings bank deposits
(5) None of these
Q.236. Which one of the following is not an objective of fiscal policy ?
(1) Economic growth (2) Economic stability
(3) Maximization of employment level (4) Regulation of financial institutions
(5) None of these
Q.237. Which of the following is not included in infrastructure in India ?
(1) Energy (2) Transport (3) Education
(4) Telecommunications (5) None of these
Q.238. The National Agricultural Insurance Scheme was started by the Government of India in _____.
(1) 1990 (2) 1995 (3) 1999 (4) 1997 (5) None of these
Q.239. Interest on national debt is included in ______.
(1) GNP (2) NNP (3) Personal Income
(4) National Income (5) None of these
Q.240. Which one of the following is not a measure of Globalisation ?
(1) Reduction in import duties
(2) Abolition of import licensing
(3) Allowing free flow of direct foreign investment
(4) Disinvestment of equity in the public sector
(5) None of these
Q.241. Which sector contributes most to the savings in India ?
(1) Public sector (2) Private corporate sector
(3) Household sector (4) Administrative sector
(5) None of these
Mahendra's GENERAL AWARENESS
Q.242. The State Bank of India was established on the recommendation of _____.
(1) Rural Banking Enquiry Committee (2) Shroff Committee
(3) All India Rural Credit Survey (4) Banking Enquiry Commission
(5) None of these
Q.243. Money Supply as defined by the Reserve Bank of India, M includes ___.
1

(1) Currency with the public


(2) Currency, demand deposits with Banks and other deposits with RBI
(3) Currency with the public and time deposits with Banks
(4) Currency with the public and deposits with National Savings Organizations
(5) None of these
Q.244. The most important source of financing the State Government's Gross Fiscal Deficit (GFD) in India
has been______.
(1) Loans from financial institutions (2) Loans from provident funds and reserve funds
(3) Loans from central government (4) Market borrowings
(5) None of these
Q.245. The system of Ad-hoc Treasury Bills in India was replaced by Ways and Means Advances with effect
from ____.
(1) 1 April , 1997
st
(2) 1 April , 1996
st
(3) 1 April , 1998
st

(4) 1 April , 1995


st
(5) None of these
Q.246. In the context of liberalization and globalization _____.
(A) Planning has no role to play
(B) Planning has an indicative role
(C) Planning has a role to play in providing safety net for the poor
(1) Only A (2) B and C (3) Only B
(4) All A, B and C (5) None of these
Q.247. Corporation tax ______.
(1) Is levied and appropriated by the states.
(2) Is levied by the union and collected and appropriated by the states.
(3) Is leveid by the union and shared by the union and the states.
(4) Is levied by the union and belongs to it exclusively.
(5) None of these
Q.248. Which of the following is an example of near money ?
(1) Small coins of 50 paise (2) Fiat Money (3) Bank Draft
(4) Treasury Bills (5) None of these
Q.249. _______ is a financial institution which mobilises savings from the people and invests then in a mix
of corporate and government securities.
(1) Commercial Bank (2) Mutual Fund (3) Unit Bank
(4) Correspondent Bank (5) None of these
Q.250. Which of the following is not a objective of Bank portfolio management ?
(1) Profitability (2) Liquidity (3) Safety
(4) Expansion (5) None of these
Q.251. A Bank which controls credit is called _____.
(1) Unit Bank (2) Central Bank (3) Commercial Bank
(4) Correspondent Bank (5) None of these
Q.252. When the government imposes physical and monetary controls to check open inflation, It is known
as ____.
(1) Mark-up Inflation (2) Demand pull Inflation (3) Creeping Inflation
(4) Suppressed Inflation (5) None of these

GENERAL AWARENESS Mahendra's


Q.253. The causes of deflation is/are _______.
(A) Lack of goods and services as campared to money supply.
(B) Low per capita income
(C) Lack of money supply as compared to supply of goods and services.
(1) A and C (2) All A, B and C (3) Only C
(4) Only A (5) None of these
Q.254. Which one of the following is not a function of Central Bank in a country ?
(1) Credit Control (2) Managing Money Supply
(3) Credit Creation (4) Regulation of Foreign Exchange
(5) None of these
Q.255. The system of Note Issue followed by Reserve Bank of India is ____.
(1) Proportional Reserve System (2) Minimum Reserve System
(3) Minimum Fiduciary System (4) Maximum Fiduciary System
(5) None of these
Q.256. A group of countries which has abolished all Traffis and also quantitative restrictions between
themselves is called ________.
(1) Economic Union (2) Customs Union (3) Common Market
(4) Free Trade Area (5) None of these
Q.257. Which one of the following is not a priority sector lending by Commercial Banks in India at present ?
(1) Agriculture (2) Small Scale Industries (3) Export
(4) Weaker Section (5) None of these
Q.258. Many times we read that a corporate entity is in the process of raising its capital base. Why is a
company required to raise money to strengthen its capital base?
(A) To finance its expansion plans
(B) To finance its diversification plans
(C) To repay its loans and borrowings
(1) Only A (2) Only B (3) Only C (4) Only A & B (5) AII A, B & C
Q.259. What are Teaser Loan rates charged by Banks ?
(1) Rate of interest in the initial period is more and it goes down subsequently.
(2) Rate of interest in the intial period is less and goes up subsequently.
(3) Fixed Rate of interest charged by Banks
(4) Floating rate of interest charged by Banks.
(5) None of these
Q.260. Which of the following institution is considered as ' soft loan window' ?
(1) IMF (2) IBRD (3) IDA (4) IFC (5) None of these
Q.261. Variable Reserve Ratio and Open Market Operations are instruments of -------------
(1) Fiscal policy (2) Monetary policy (3) Budgetary policy
(4) Trade policy (5) None of these
Q.262. In which year SMERA (Small & Medium Enterprises Rating Agency) was established ?
(1) 2002 (2) 2005 (3) 2008 (4) 2001 (5) None of these
Q.263. The Bank rate policy is a component of ------------
(1) Fiscal policy (2) Monetary policy (3) Trade policy
(4) Foreign policy (5) None of these

Mahendra's GENERAL AWARENESS


Q.264. Consider the following statements regarding the features of the Indian money market----
(A) It finds avenues for profitable investment for short -term surplus
(B) It opens the door for the Commercial Banks to occupy a strategic position in regulating
money market.
(C) It provides short- term funds for Banks.
Which of the statements given above is/are correct ?
(1) Only A (2) Only B (3) A and C (4) All A,B and C (5) None of these
Q.265. If the Reserve Bank of India sells securities in the market it will result in -------------------
(1) An immediate change in the Bank rate (2) A fall in the market rate of interest
(3) An increase in loans to Bank customers (4) A reduction in Bank deposits
(5) None of these
Q.266. The majority of Central Government enterprises belong to the ----------
(1) Public corporations (2) Public limited companies
(3) Private limited companies (4) Departmental organisations
(5) None of these
Q.267. The Central Bank can decrease the Bank credit component of the money supply by ------------
(1) Lowering the cash reserve requirements (2) Increasing the Bank rate
(3) Lowering the Bank rate (4) Buying of government securities
(5) None of these
Q.268. Parallel Economy is also referred as ----------
(1) Grey Market (2) Black Market (3) Black Economy
(4) Both 1and 2 (5) None of these
Q.269. Which of the following was the first commercial Bank of limited liability managed by Indians ?
(1) Punjab National Bank (2) Oudh Commercial Bank
(3) Imperial Bank of India (4) Bank of Baroda
(5) None of these
Q.270. --------------- is an agreement where by an issuing Bank at the request of the importer (Buyer)
undertakes to make payment to the exportor (Beneficiary) against stipulated documents.
(1) Bills of exchange (2) Letter of exchange (3) Letter of credit
(4) Bill of entry (5) None of these
Q.271. What are Time Liabilities ?
(1) The liabilities which Bank have to pay on demand
(2) The liabilities which Bank have to pay after specific time period.
(3) The liabilities which Bank have to pay to the Reserve Bank of India
(4) The liabilities which Bank have to pay to the central government
(5) None of these
Q.272. Regional Rural Banks are classified under ---------------
(1) Land mortgage Banks (2) Co-operative Banks (3) Commercial Banks
(4) Public sector Banks (5) None of these
Q.273. As we all know that Reserve Bank of India manages the circulation of currency. In this context which
of the following decides the currency denomination value ?
(1) Planning Commission (2) Reserve Bank of India Itself
(3) Government of India (4) Finance Commission
(5) None of these
Q.274. The country's First ever socio-economic and caste census to identify people living below poverty
line was recently started from a tribal village Sankhola in _____
(1) Tripura (2) Assam (3) West Bengal (4) Maharashtra (5) None of these
GENERAL AWARENESS Mahendra's
Q.275. Which of the following is India's one of the major Microfinance Institution ?
(1) Grameen (2) Bantra (3) Brae (4) Spandana (5) None of these
Q.276. When does the Central Bank increases the minimum statutory cash reserve ratio of the Commercial
Banks ?
(A) When the economy is in deflationary condition
(B) When the economy is experiencing inflation
(C) When the Central Bank aims credit contraction.
(1) A and C (2) All A, B and C (3) B and C (4) Only B (5) None of these
Q.277. The primary objective of selective credit control is _______.
(1) To raise the cost of credit for all purposes.
(2) To decrease the total supply of credit in the economy.
(3) To regulate total Bank credit and general level of interest rates.
(4) To influence allocation of credit among different borrowers and users.
(5) None of these
Q.278. Consider the following measures.
(A) Changing the Bank rate
(B) Open market operation
(C) Increasing the Cash-Reserve Ratio
Which of the measures given above are taken by the Reserve Bank of India to control inflation in
India ?
(1) A and B (2) All A, B and C (3) A and C
(4) Only C (5) None of these
Q.279. Bank rate refers to the _____.
(1) Interest rate at which Commercial Banks accept deposits from the public.
(2) Rate at which Central Bank rediscounts bills of exchange of Commercial Banks.
(3) Prime Lending Rate of Commercial Banks
(4) Interest rate at which Commercial Banks lend to the customers
(5) None of these
Q.280. Which of the following would reduce the credit creation capacity of a Commercial Bank ?
(A) Deposits with the Central Bank (B) Time and Demand deposits (C) Cash in hand
(1) A and C (2) Only B (3) A and B
(4) All A, B and C (5) None of these
Q.281. Which one of the following items is not included in the current account of India's balance of
payments ?
(1) Short-term commercial borrowings (2) Non-monetary gold movements
(3) Investment income (4) Transfer payments
(5) None of these
Q.282. The Reserve Bank of India placed the greatest reliance on which of the following measures of credit
control for maintaining price stability during the last ten years ?
(1) The Bank Rate (2) Open Market Operations
(3) Cash reserve requirements (4) Statutory liquidity requirements
(5) None of these
Q.283. Which one of the following groups of items is included in India's foreign exchange reserves ?
(1) Foreign-currency assets, Special Drawing Rights (SDRs) and Loans from foreign countries.
(2) Foreign-currency assets, Gold holdings of RBI and Special Drawing Rights (SDRs)
(3) Foreign-currency assets, Loans from the World Bank and Special Drawing Rights (SDRs)
(4) Foreign-currency assets, Gold holdings of RBI and Loans from the World Bank.
(5) None of these

Mahendra's GENERAL AWARENESS


Q.284. If there is an inflationary trend in the economy what would be the trend in the pricing of the Bank
products ?
(1) Constant trend
(2) Increasing trend
(3) Decreasing trend
(4) There is no relevance of the inflation in pricing of the banking products
(5) None of these
Q.285. For a closed economy having no foreign trade which one of the following is correct ?
(1) GDP > GNP (2) GDP < GNP (3) GDP = GNP
(4) GDP < GNP (5) None of these
Q.286. The ability of a commercial Bank to increase its loans and Investment depends on its _______.
(1) Holding of government securities (2) Cash position
(3) Excess Cash reserves (4) Outstanding loans and investments
(5) None of these
Q.287. 'Economic Development' is a _______.
(A) Continuous process
(B) Short-term process
(C) Long term process
(1) Only C (2) A and C (3) Only A (4) A and B (5) None of these
Q.288. Which of the following is not a feature of Microfinance ?
(1) It provides gainful financial services to unemployed and low-income individuals.
(2) It promotes saving of money by individuals.
(3) Interest rates are generally lower than those offered by normal banks.
(4) Micro finance operations make profit from the poor.
(5) None of these.
Q.289. Which of the following is not the part of the scheduled banking structure in India ?
(1) Money Lenders (2) Public Sector Banks
(3) Private Sector Banks (4) Regional Rural Banks
(5) State Co-operative Banks
Q.290. Which of the following cannot be called as a value Added service offered by a Bank?
(1) Special accounts for poor sections of the society
(2) Accident insurance cover
(3) Instant Credits of Outstation Cheques
(4) Free cheque book
(5) All are value Added services
Q.291. Opening the Saving Bank Account of a minor girl will be called as which of the following in Banking
terminology ?
(1) Retail Banking (2) Merchant Banking (3) Institutional Banking
(4) Social Banking (5) Corporate Banking
Q.292. Which of the following terms is NOT used in Banking Sector?
(1) SLR (2) NPA (3) Credit Rating(4) Fixed Deposit (5) PURA
Q.293. Banking Sector will fall under which of the following sectors?
(1) Agriculture Sector (2) Service Sector (3) Manufacturing
(4) Industrial Sector (5) None of these
Q.294. Which of the following is NOT a measure of the Risk Management in Banks ?
(1) CRR (2) RTGS (3) SLR
(4) Deposit Insurance (5) All are the measures of risk management

GENERAL AWARENESS Mahendra's


Q.295. Which of the following Acts was framed to deal more effectively specially with the problem of Non-
Performing Assets in banking system?
(1) SARFAESI ACT (2) Banking Regulation Act.
(3) Foreign Exchange Management Act (4) Industrial Dispute Act.
(5) None of these
Q.296. Many times we read a term CBS used in banking operation. What is the full form of the letter 'C' in the
term 'CBS'?
(1) Core (2) Credit (3) Continuous
(4) Complete (5) None of these
Q.297. Name any E-Banking service which is based on voice processing facility ?
(1) Any time banking (2) Tele-banking (3) On-line banking
(4) Any where banking (5) None of these
Q.298. Non Performing Assets are those assets which are overdue for a period of more than ?
(1) 90 days in interest and/or installment in term of loan.
(2) 90 days out of order in case of over draft/cash credit. Bills purchased and Bills discounted.
(3) 2 harvest seasons for advances for cultivation of short duration crops.
(4) All of the above
(5) None of these
Q.299. _______has been set up for exchange of credit information among its members ?
(1) DICGCI (2) CIBIL (3) ECGL (4) EXIM Bank (5) None of these
Q.300. Basel - II norms are associated with sector.
(1) Banking (2) Insurance (3) Share
(4) All of the above (5) None of these
Q.301. Which among the following is also known as Pacific Rim trade IBSA ?
(1) APEC (2) ASEAN (3) SAARC (4) IBSA (5) IBRD
Q.302. Which one of the following will setup core banking infrastructure for rural banks ?
(1) RBI (2) NABARD (3) SIDBI (4) IBA (5) None of these
Q.303. Who is the Ex-Officio chairmain of Niti Aayog ?
(1) The President (2) The Finance Minister (3) The Prime Minister
(4) The Chief of CSO (5) None of these
Q.304. Which of the following is correct regarding the Gross Domestic Saving in India?
(1) Contribution of household sector is the largest
(2) Contribution of govt. sector is the largest
(3) Contribution of corporate sector is the largest
(4) All of the above
(5) None of these
Q.305. Gilt-edged market means________
(1) Bull market (2) Market of govt. securities
(3) Market of guns (4) Market of Pure metals
(5) None of these
Q.306. Who is the Chairman of ISRO at present?
(1) Pradeep Kumar (2) K. Radhakrishnan (3) A.S. Kiran Kumar
(4) Rakesh Sood (5) None of these
Q.307. What do you understand by "Para Banking" services ?
(1) Eligible financial services rendered by bank
(2) Utility services provided by bank
(3) Services provided through business correspondent
(4) Services provided to armed force personnel
(5) None of these

Mahendra's GENERAL AWARENESS


Q.308. In the context with Investment what are A, AA+ and AAA ?
(1) Stock exchange Index (2) Credit ratings
(3) Market stability Index (4) Profit equity Index
Q.309. High powered money is _________
(1) Bank's reserves at central banks
(2) All loans and advances of banks
(3) Money held by banks
(4) Currency held by public and reserves with the central bank
(5) None of these
Q.310. A significant decrease in the demand for loans will force Banks to _________.
(1) Adjust their portfolios (2) Sell securities to the central Bank
(3) Resort to borrowings (4) Raise the prime lending rate
(5) None of these
Q.311. The process of credit creation has the effect of ______
(1) Raising the real national income
(2) Raising the real wealth of a country
(3) Increasing the supply of money
(4) Decreasing the national debt
(5) None of these
Q.312. Which of the following is not a function of commercial banks ?
(1) Acceptance of deposits
(2) Purchase and sale of foreign exchange
(3) Discounting bills of exchange
(4) Controlling credit expansion
(5) None of these
Q.313. Which of the following is generally referred to as a 'Broader' measures of money supply ?
(1) M1 (2) M
2 (3) M
3 (4) M
4 (5) None of these
Q.314. The RBI has helped to finace India's foreign Trade through ____
(1) NABARD (2) EXIM Bank (3) SBI (4) IDBI (5) None of these
Q.315. A Maharatna company can invest which of of the following amounts independently for a project ?
(1) Rs 20.000 crore (2) Rs 12,000 crore (3) Rs 10,000 crore
(4) Rs 5000 crore (5) None of these
Q.316. The Tier - I Capital of Banks does not includes __________
(1) Paid up capital (2) Statutory reserves
(3) Revaluation reserves (4) Investment Fluctuation reserves
(5) None of these
Q.317. Which of the following scheme is related with Financial Inclusion ?
(1) SABLA (2) Swavalamban (3) Swabhiman
(4) Adhaar (5) None of these
Q.318. With reference to Banking what is PCR ?
(1) Profit Credit Ratio (2) Provision Credit Ratio
(3) Provision Coverage Ratio (4) Provision Cash Ratio
(5) None of these
Q-319. Goods and Services Tax (GST), would replace which of the following tax/taxes ?
(A) Income Tax (B) Corporate Tax (C) Value Added Tax (VAT)
(1) Only A (2) Only B (3) Only C (4) All A, B and C (5) None of these

GENERAL AWARENESS Mahendra's


Q-320. Agricultural taxation in India is difficult because of -
(1) Scattered nature of land holdings
(2) Seasonality of production
(3) The fact that agriculture is not taxed any where in the world
(4) Increasing overhead cost of tax collection and the lack of political will
(5) None of these
Q-321. Kisan Credit Card Scheme (KCC) was introduced in 1998-99 to improve the credit delivery to
farmers. Which one of the following does not operate this scheme ?
(1) NABARD (2) Co-operative Banks (3) Regional Rural Banks
(4) Schedules Commercial Banks (5) None of these
Q-322. Which one of the following is a source of non-tax revenue for Government of India ?
(1) Import duty on cars (2) Octroi at cheek points on roads
(3) Entrance fee on museums (4) Excise duty on beverages
(5) None of these
Q-323. Many a times we read in newspapers a term CBS. What does B denotes in the term 'CBS' ?
(1) Business (2) Banking (3) Base (4) Bond (5) None of these
Q-324. The back bone of Indian economy is/are
(A) Industry (B) Agriculture (C) Export
(1) Only A (2) Only B (3) A and C
(4) All A, B and C (5) None of these
Q-325. Small loans provided by the Banks to very poor families without any collateral security is popularly
known as
(1) Personal finance (2) Project finance (3) Macro finance
(4) Micro finance (5) None of these
Q-326. "One Family One Bank" tagline is associated with a PSB Bank. The Public Sector Bank (PSB) in the
Question is
(1) Canara Bank (2) Bank of Maharashtra (3) Bank of India
(4) Corporation Bank (5) None of these
Q-327. Co-operative movement in which of the following section of the indian economy has been highly
successful ?
(1) Small scale sector (2) Milk production (3) Transport sector
(4) Energy sector (5) None of these
Q-328. An indicator currently used by United nations development programme (UNDP) for measure ment
of the quality of people's life is known as
(1) Human development Index (2) Average expectancy of life
(3) Per capita income (4) Standard of living index
(5) None of these
Q-329. Which of the following agencies was set up to promote new technological inputs in rural sector ?
(1) CAPART (2) HUDCO (3) SIDBI (4) IDBI (5) None of these
Q.330. What are the objective taken by NABARD to improve the Rural Banking infrastructure ?
(1) For the development of education in rural area
(2) For the development of health of women and children
(3) For availability of food grains
(4) For insuring Prosperity and Progress integrated development of rural areas
(5) None of these

Mahendra's GENERAL AWARENESS


Q.331. "Open Market Operation" are controlled by -----.
(1) World Bank (2) IMF (3) RBI (4) Finance Ministry
Q.332. Which are following statement(s) is/are correct definition of 'payment balance' in economy terms?
(A) This is transaction records among the country in which payment is made in currency other
than US dollar
(B) This is a transaction record between specific country and all countries.
(C) This is record of export/import of a country and difference of amount in US$
(1) Only A (2) Only B (3) Only C (4) Only A and C (5) None of these
Q.333. RBI has permitted non banking finance companies (NBFCs) to enter the _____ business through
the joint venture route.
(1) Mutual Fund (2) Insurance (3) Universal Banking
(4) Merchant Banking (5) None of these
Q.334. Which of the following is not a part of the World Bank?
(1) IBRD (2) ADB (3) IDA (4) IFC (5) MIGA
Q.335. Which of the following statements is/are true about the shareholding in the Regional Rural Bank
(RRB)?
A. Central govt. holds 50% stake.
B. RBI holds 50% stake.
C. State government holds 15% stake.
D. Sponsored bank holds 35% stake.
(1) Only A & B (2) Only B, C & D (3) Only A, C & D
(4) Only A & D (5) None of these
Q.336. Open market operations of RBI refers to -
(1) Buying and selling of securities (2) Trading in shares
(3) Auctioning of treasury bills (4) Selling & purchasing of goods
(5) None of these
Q.337. Which of the following is called "paper gold"?
(1) Euro (2) Dollar (3) Yuan (4) SDR (5) None of these
Q.338. Which of the following is/was Not included in the agenda set for the Banking Reforms in India?
(1) Dismantling of administered interest rates
(2) Measure to strengthen risk management
(3) Promulgation of SARFAESI Act
(4) Promotion of the concept of easy credit to all with a guaranteed subsidy from the government
(5) None of these
Q.339. In respect of which one of the following areas, Y. H. Malegam Committee has submitted its
recommendations?
(1) Interest on Micro Finance loans (2) Teaser loans
(3) Rural development (4) Advances to agriculture
(5) None of these
Q.340. The Agriculture Insurance Company was launched with the help of-
(1) RBI (2) SIDBI (3) NABARD (4) ECGC (5) None of these
Q.341. Deflation refers to -
(1) General decline in price of various commodities and services
(2) Price decline in specific sectors.
(3) Price decline in specific commodities.
(4) Sharp and sudden decline in foreign exchange reserves.
(5) None of these

GENERAL AWARENESS Mahendra's


Q.342. Which of the following agencies/organizations in India maintains the Micro Finance Development
and equity fund?
(1) Confederation of Industries in India. (2) Indian Bank's Association
(3) Small industries Development Bank of India (4) Reserve Bank of India
(5) None of these
Q.343. In the term of economics, the recession occuring two times with a small gap in between is known as -
(1) Double Deflation (2) Deflation (3) Deep Recession
(4) Double Dip Recession (5) None of these
Q.344. Which of the following terms is not used in Economics?
(1) Balance of Payment (2) Call Money (3) National Debt
(4) Elasticity of Demand (5) Boyle's Law
Q.345. The Reserve Bank of India is regularly revising upwards the repo and reverse repo rates. Why is
this being resorted to?
(1) To check inflationary pressures in the economy (2) To Curb growth of black money
(3) To encourage rise in interest rates on deposits. (4) To make bank loan costiler
(5) None of these
Q.346. 'Sub Prime Lending' is a term applied to the loans made to-
(1) Those borrowers who do not have a good credit history.
(2) Those who wish to take loan against the mortgage of tangible assets.
(3) Those who have a good credit history and are known to bank since 10 years.
(4) Those borrowers who are most preferred customers of the bank.
(5) None of these
Q.347. Expand the term ALM as used in Banking/Finance sector?
(1) Asset Liability Mismatch (2) Asset Liability Maturity
(3) Asset Liability Management (4) Asset Liability Manpower
(5) None of these
Q.348. Basel - II norms are associated with which of the following aspects of the banking industry?
(1) Risk Management (2) Manpower Planning
(3) Retirement benefits for the employees (4) Corporate governance
(5) None of these
Q.349. What is stagflation?
(1) Inflation with growth (2) Deflation with growth
(3) Inflation after deflation (4) Inflation with recession
(5) None of these
Q.350. SEBI is a -
(1) Statutory body (2) Advisory body (3) Constitutional body
(4) Non-Statutory body (5) None of these
Q.351. Which statement is correct for the Planning Commission of India?
(1) It is not defined in Indian Constitution
(2) Its members and Deputy Chairman do not have any fixed working duration
(3) Its members do not require any minimum qualification.
(4) All above are true
(5) None of these
Q.352. As per the new guidelines issued by RBI, the minimum required capital to set up a bank by
corporates is set at ________.
(1) Rs. 600 crore (2) Rs. 700 crore (3) Rs. 400 crore
(4) Rs. 500 crore (5) None of these
Mahendra's GENERAL AWARENESS
Q.353. The economy of India is/are facing which of the following problem/problems now a days?
A. Huge trade deficits.
B. Disequilibrium in balance of payments.
C. High inflation and high food and commodity prices.
(1) Only A (2) Only B (3) A and C (4) Only C (5) None of these
Q.354. The primary monetary policy technique employed by the Reserve Bank of India is ________.
(1) Reserve requirements (2) Open market operations
(3) Discount policy (4) Margin requirements
(5) None of these
Q.355. When a bank writes off a loan as bad, its ________.
(1) Total assets and total liabilities decreases by that amount.
(2) Total liabilities and capital decreases by that amount.
(3) Total assets and capital decreases by that amount.
(4) Total assets, total liabilities and capital decreases by the amount.
(5) None of these
Q.356. Which of the following organisations maintains CRR?
(1) SEBI (2) RBI (3) Ministry of Finance
(4) NABARD (5) None of these
Q.357. Many a times we read in financial newspapers about channel financing. 'Channel Financing' is/are
associated with ________.
(A) SME lending (B) Supply chain finance (C) Retail lending
(1) Only A (2) Only B (3) A and C (4) Only C (5) None of these
Q.358. Invisible export means export of ________.
(1) Services (2) Prohibited goods (3) Unrecorded goods
(4) Goods through smuggling (5) None of these
Q.359. Which of the following is not the part of prudential guidelines issued by Reserve Bank of India?
(1) Provisioning (2) Loan recovery rules (3) Asset classification
(4) Income recognition (5) None of these
Q.360. Payment of a cheque cannot be made on a cash counter of bank, if the cheque is ___
(1) Bearer (2) cross (3) order (4) All of above (5) None of these
Q.361. Bank draft is a _______
(1) letter from the bank
(2) cheque which a bank draws itself
(3) Instruction to a banker to collect a customer's debt.
(4) Instruction not to honour a stop payment
(5) None of these
Q.362. Which is not an example of near money?
(1) Bill of exchange
(2) Govt. bonds and debentures
(3) Equity shares of Ranbaxy company limited
(4) Treasury bills of the Government of India
(5) None of these
Q.363. Demand pull inflation can be caused by money factors including _______
(1) a fall in the consumption expenditure
(2) a sharp increase in the unemployment
(3) a steep reduction in the direct taxation
(4) an increase in the income tax
(5) None of these
GENERAL AWARENESS Mahendra's
Q.364. Which of the following is not a function of the commercial Bank ?
(1) Acting as a lender of last resort.
(2) Lending to the private and public sectors
(3) The provision of a cheque system for setting debts
(4) The provision of safe deposit facilities
(5) None of these
Q.365. A customer does not require a bank account to obtain _________
(1) a loan (2) an overdraft (3) a cheque
(4) a banker's draft (5) None of these
Q.366. Which of the following banks were merged into to form Imperial Bank of India?
(1) Bank of Bengal (2) Bank of Madras (3) Bank of Bombay
(4) Bank of Gujarat (5) 1, 2 and 3
Q.367. Which bank launched Indias first loan disbursement card?
(1) SBI (2) UTI (3) IDBI (4) ICICI (5) None of these
Q.368. For which of the following banks, did RBI introduce the scheme protected disclosure?
(1) Private Banks (2) Co-operative Banks
(3) Foreign Banks (4) 1 and 3
(5) 2 and 3
Q.369. 'The Decade of Action for Road safety' campaign by ___________ was launched in India recently.
(1) World Bank (2) UNO (3) WTO (4) SAARC Nations (5) None of these
Q.370. External Commercial Borrowings is a part of _________
(1) Balance of Payment (2) Balance of Trade (3) Current Account
(4) Capital Account (5) None of these
Q.371. What is the current duration of New Foreign Trade Policy?
(1) 2004-2009 (2) 2009-2014 (3) 2010 -2015 (4) 2007-2012 (5) None of these
Q.372. Adequacy of bank's liquidity position depends upon ______
(1) Source of funds (2) Anticipated future funding needs
(3) Present and future earnings capacity (4) All of above
(5) none of these
Q.373. RBI's open market operation transactions are carried out with a view to regulate _______
(1) Liquidity in the economy (2) Prices of essential commodities
(3) Inflation (4) Borrowing power of banks
(5) All of above
Q.374. Currency swap is an instrument to manage _________
(1) Currency risk (2) Currency & interest rate risk
(3) Cash flows in different currency (4) Interest rate risk
(5) All of above
Q.375. BASEL - III will be implemented from _____.
(1) 2012 (2) 2013 (3) 2014 (4) 2015 (5) None of these
Q.376. Which of the following in not included in M2?
(1) Currency and coins (2) Corporate bonds held by firms and individuals
(3) Demand deposits (4) Money market mutual fund shares
(5) none of these

Mahendra's GENERAL AWARENESS


Q.377. Contractual savings institutions include ______
(1) Commercial banks & thrifts
(2) Finance companies and mutual funds
(3) Life insurance companies and pension funds
(4) All of above
(5) None of these
Q.378. In which year did the United Nations General Assembly adopted the World Environment Day?
(1) 1970 (2) 1972 (3) 1981 (4) 1985 (5) None of these
Q.379. Foreign bank means a bank ___________
(A) That has been incorporated outside India
(B) That has its 51% or more branches outside India
(C) Whose more than 51% equity capital is owned by foreigner.
(1) Only A (2) A and B (3) B and C (4) A, B and C (5) None of these
Q.380. RRB's were established on the recommendations of _________.
(1) Narsimham committee (2) N. L. Dantwala committee
(3) B.Shivraman Committee (4) S. M. Kelkar committee
(5) none of these
Q.381. Contribution of Central Government, State Government and Sponsored Bank in paid up capital of a
Regional Rural Banks is in the ratio of ________
(1) 50: 15: 35 (2) 60: 25: 15 (3) 60: 20: 20 (4) 50: 25: 25 (5) None of these
Q.382. Which of the following terms indicates a mechanism used by commercial banks for providing credit
to the government?
(1) Cash Credit Ratio (2) Debt Service Obligation
(3) Liquidity Adjustment Facility (4) Statutory Liquidity Ratio
(5) None of these
Q.383. In the opinion of the Governor of Reserve Bank of India, which one of the following is the reason
owing to which indias inflation is accelerating ?
(1) Excess liquidity in market. (2) Speculation in essential goods
(3) Higher Food cost (4) Commodities Futures
(5) None of these
Q.384. The tagline of Indian Overseas Bank is -
(1) Good people to grow with (2) Good people to banks with (3) Pure banking
(4) Only Banking nothing else (5) None of these
Q.385. The organisation is responsible for calculating National Income is -
(1) CSO (2) NSSO (3) RBI (4) SBI (5) None of these
Q.386. RBI was Nationalized on -
(1) 1 Apri 1949
st
(2) 1 Jan 1949
st
(3) 1 September 1949
st

(4) 1 December 1949


st
(5) None of these
Q.387. The minimum limit with RTGS is _________
(1) 2 lakh (2) 3 lakh (3) 4 lakh (4) 6 lakh (5) None of these
Q.388. The idea of Lokpal has emanated from the office of Ombudsman, which is prevalant in ______
countries ?
(1) Latin Amircian (2) South East Asian (3) Scandivanian
(4) European (5) African
Q.389. The tax on Import and Export is known as -
(1) Income Tax (2) Trade Tax (3) Custom duty
(4) Commercial Tax (5) None of these
GENERAL AWARENESS Mahendra's
Q.390. The National Rural Livelihood Mission (NRLM) was launched recently.
Which of the following statement is/are correct regarding NRLM ?
(A) NRLM seeks to bring the rural poor under the self help group (SHG) network.
(B) Servics of Business correspondent
(C) NRLM will facilitate access of bank credit to those covered under the mission
(1) Only A (2) Only A & B (3) Only B and C
(4) Only A and C (5) None of these
Q.391. The rate of interest on term deposits in banks is stipulated by which of the following ?
(1) RBI (2) Government of India
(3) Indian Banks Association (4) Banking codes and standards board of India
(5)Bank Itself
Q.392. Chief of RBI is known as ----------
(1) President (2) Chairman (3) Managing director
(4) Director (5) None of these
Q.393. In order to streamline the functioning of MFI sector the RBI has appointed a committee. The
committee was headed by ------------
(1) Dr. Y.V. Reddy (2) Shri M.V. Nair (3) Sri Y.H. Malegam
(4) Dr. D. Subbarao (5) None of these
Q.394. Which of the following is/are the depositories in India handling dematerialized shares ?
(A) National payment corporation of India
(B) Central depository services limited
(C) Bombay stock exchange (BSE)
(1) Only A & B (2) Only A (3) Only B
(4) Only C (5) None of these
Q.395. What is the total number of stock exchanges in India ?
(1) 21 (2) 22 (3) 23 (4) 24 (5) None of these
Q.396. Resident foreign currency account can open only for---------
(1) Scheduled commercial banks (2) Public sector banks
(3) Private sector Banks (4) Regional Rural banks
(5) None of these
Q.397. First bank operated by Indians on limited liability basis was -------------
(1) Punjab National Bank (2) Oudh Commncial Bank
(3) Allahabad Bank (4) Imperial Bank of India
(5) None of these
Q.398. Body which regulates microfinance is -
(1) RBI (2) Indian Banks Association (3) NABARD
(4) SIDBI (5) None of these
Q.399. Govt's Income from issuance of note is part of -----------
(1) Tax Revenue Receipts (2) Non tax Revenue
(3) Capital Receipts (4) Non debt capital Receipts
(5) None of these
Q.400. The Rajiv Gandhi Action plan, put forwarded in 1988 is related to which among the following ?
(1) Environmental Protection (2) Nuclear Disarmament
(3) Information Technology (4) Multilateral Trade
(5) None of these

Mahendra's GENERAL AWARENESS


Q.401. Which of the following describes the term 'legal tender?
(1) Any object legally used as money in a country
(2) Money which cannot legally be refused in discharge of a debt
(3) Bank notes where they are in legal circulation
(4) Notes or coins other than the counterfeited ones
(5) None of these
Q.402. What is the share of sponsor bank in the capital of RRB ?
(1) 35% (2) 49% (3) 51%
(4) 71% (5) None of these
Q.403. The word NELP is associated with _____
(1) Discovery of Oil & Natural Gas (2) National Highway extension
(3) Communication (4) Space research
(5) Power sector
Q.404. What is the base year for WPI (Whole Sale Price Index) ?
(1) 2004-05 (2) 2003-04 (3) 2002-03
(4) 2005-06 (5) None of these
Q.405. RBI increases Repo Rate to control which economic situation of economy ?
(1) Deflation (2) Inflation (3) Recession
(4) Stagflation (5) None of these
Q.406. A limited company is ________
(1) In which shareholders possess the ownership limited to their paid up capital
(2) In which shares are issued
(3) A company of Govt. ownership
(4) A registered company
(5) None of these
Q.407. What is the objective of Kisan call centre ?
(1) To ensure the supply of fertilizer to farmers on suitable rates
(2) Distribution of HYV seeds
(3) To provide agriculture credit at concessional rate
(4) To provide consultancy service related to agriculture
(5) None of these
Q.408. Moral Suasion is -------
(1) A term descriptive of corrupt practices of many banks
(2) A term applied to banks that are conservative in their banking practices
(3) Persuasion of banks to adhere to central bank desires
(4) It denotes conduct or character that is bad and vice
(5) None of these
Q.409. Which of the following rates is not determined by RBI ?
(1) Bank Rate (2) CRR (3) SCR (4) PLR (5) MSF
Q.410. Which of the following were called the Presidency Bank ?
(A) Bank of Bengal , Bank of Madras, Bank of India.
(B) Bank of Bengal , Bank of Bombay, Bank of Madras
(C) State Bank of India, Punjab National Bank, Allahabad Bank
(1) Only A (2) Only B (3) Only C (4) Both A & B (5) Only A & C
Q.411. When was New Bank of India merged with Punjab National Bank ?
(1) 4 sept 1993
th
(2) 4 sept 1983
th
(3) 1 July 1935
st

(4) 2 October 1995


nd
(5) 2 April 1993
nd

GENERAL AWARENESS Mahendra's


Q.412. Which of the following deposits is not accepted by commercial banks?
(1) Fixed Deposit (2) Demand Deposit
(3) Saving Deposit (4) Public Account Deposit
(5) None of these
Q.413. Which of the following in not the method of measuring National Income ?
(1) Product method (2) Income Method (3) Consumption Method
(4) Production method (5) None of these
Q.414. What is the total No. of PSEs companies listed in Navratna status ?
(1) 9 (2) 10 (3) 16 (4) 12 (5) None of these
Q.415. Who is the chairman of Indian oil corporation Ltd ?
(1) N.C. Jha (2) B. Ashok (3) A.K. Hazarika
(4) D. Subbarao (5) Chandra Sekhar Verma
Q.416. The International monetary fund (IMF) is an organization of ________ countries
(1) 237 (2) 126 (3) 188 (4) 430 (5) 200
Q.417. According to RBI Governor, which one of the following is a major challenge to its monetary policy ?
(1) Policies of World Trade Organisation
(2) High cost of imports
(3) Fiscal deficit
(4) Inflation and supportive recovery of Economy
(5) None of these
Q.418. Which one of the following denomination of plastic notes is proposed to be introduced by RBI ?
(1) 5 (2) 10 (3) 20 (4) 50 (5)100
Q.419. Which of the following documents is/are generally sought for Housing loan approval?
(A) Residence proof
(B) Latest salary slip
(C) Last 6 months bank statements
(1) Only A (2) Only B (3) All A, B and C
(4) A and C (5) None of these
Q.420. Whether loss-making banks can make donations?
(1) Upto Rs. 5 lakh only (2) No (3) Upto Rs. 8 lakh only
(4) Upto Rs. 2 lakh only (5) None of these
Q.421. The Basic Banking Services of a Bank include/includes _______.
(A) Services relating to deposit accounts
(B) Remittance facilities
(C) Collection facilities
(1) Only A (2) All A, B and C (3) Only B (4) Only C (5) None of these
Q.422. Which of the following are not insured by the Deposit Insurance and Credit guarantee Corporation
(DICGC)?
(1) Commercial Banks (2) Local Area Banks
(3) Regional Rural Banks (4) Primary Co-operative Societies
(5) None of these
Q.423. In which of the following sector/sectors FDI is not allowed in India, under the automatic route as well
as the government route?
(A) Gambling and Betting
(B) Atomic Energy
(C) Retail Trading (Except single brand product retailing)
(1) All A, B and C (2) Only A (3) Only B (4) Only C (5) None of these
Mahendra's GENERAL AWARENESS
Q.424. In addition to cash dispensing, the other services/facilities available at ATMs includes _____.
(A) Account information
(B) Purchase of Re-load Vouchers for mobiles
(C) Loan account enquiry
(1) Only A (2) Only B (3) Only C (4) All A, B and C (5) None of these
Q.425. An EEFC (Exchange Earner's Foreign Currency) account can be held only in the form of ____.
(1) Saving Account (2) Current Accoun (3) Fixed Deposit Account
(4) Nostro Account (5) None of these
Q.426. Reserve Bank of India relies on data submitted by Banks and quality of data is of great importance.
In order to meet the need for correct and consistent data, RBI has initiated the project on ADF. What
is the full form of ADF?
(1) Automated Data Flow (2) Automatic Document Feeder
(3) Automatically Defined Function (4) Automatic Direction Finder
(5) None of these
Q.427. Consider the following:-
(A) Public Sector Undertakings
(B) Private Sector Companies
(C) Ministries and Departments
Which among the above are eligible for Indira Gandhi Rajbhasha Awards for outstanding
achievements in the implementation of Hindi ?
(1) A & C (2) A & B (3) B & C (4) A, B & C (5) None of these
Q.428. Where is located Indias first operational special economic zone (SEZ) ?
(1) Ahmedabad (2) Surat (3) Jaipur (4) Indore (5) None of these
Q.429. Who among the following are the beneficiaries of the Reverse Mortgage Scheme. ?
(1) Government Employees (2) Senior Citizens
(3) Unemployed Persons (4) Persons of BPL Category
(5) None of these
Q.430. Under which among the following acts, Hawala transactions are banned in India ?
(1) FERA (2) FEMA (3) FRBMA (4) AFSPA (5) None of these
Q.431. Consider the following:
(A) Liquidity Adjustment Facility
(B) Devaluation of Currency
(C) Open Market Operations
Which among the above are Fiscal Measures?
(1) A & B (2) B & C (3) A & C (4) Only B (5) None of these
Q.432. Which among the following terms is commonly not associated with Budgets in India?
(1) Outcome Budget (2) Gender Budget (3) Austerity Budget
(4) Gross Budgetary Support (5) None of these
Q.433. Which of the following is not helpful in controlling the money supply ?
(1) Changes in reserve requirements (2) CRR
(3) Open market policy (4) Bank rate
(5) None of these
Q.434. Which of the following is a form of "Small Savings Bank" popular among the poor or children ?
(1) Core Banking (2) Credit Banking (3) Debit Card
(4) Merchant Banking (5) Piggy Banking

GENERAL AWARENESS Mahendra's


Q.435. Which of the following statement(s) is/are not true about overdraft ?
(A) It occurs when the account balance reaches the level of minimum balance.
(B) It is a form of credit extended by a creditor when the account balance reaches zero.
(C) Overdrafts allow bank clients to withdraw money even when there are no funds in the account.
(1) Only A and B (2) Only B (3) Only A and C
(4) Only A (5) Only B and C
Q.436. Which statement correctly describes Bridge financing' ?
(1) Finance provided to bridge the gap between poor and the rich.
(2) Bridge financing begins at the end of the first loan and ends with the start of the second loan.
(3) Bridge financing ends at the start of the first loan and starts with the end of the second loan.
(4) Finance provided to bridge the gap between Rural and the Urban people.
(5) Bridge financing is provided to fill the social gaps.
Q.437. A very large loan extended by a group of small banks to a single borrower, especially corporate
borrowers is known as _______.
(1) Syndicated Loan (2) Group Loan (3) Standard Loan
(4) Organised Loan (5) None of these
Q.438. The public debt office function under the control of __________________
(1) Ministry of Finance (2) Reserve Bank of India (3) SEBI
(4) IRDA (5) None of these
Q.439. Consider the following-
(A) Goodwill
(B) Trademarks, Copyrights or Patents
(C) Government Securities
(D) Tax Refunds
(E) Certificates of Deposit
Which of these are the examples of liquid assets ?
(1) A, B, and C (2) C and D (3) C, D and E (4) A, B and E (5) All of these
Q.440. Which of the following describes Hot Money ?
(A) Flow of funds (or capital) from one country to another in order to earn a short-term profit on
interest rate differences and/or anticipated exchange rate shifts.
(B) Speculative capital flows that move very quickly in and out of markets.
(C) Capital that is responsible for stabilizing domestic money market.
(1) Only A and C (2) Only B (3) Only A and B
(4) Only A (5) None of these
Q.441. When the exchange rate is adjusted so that an identical good in two different countries has the same
price when expressed in the same currency. It is called______.
(1) Purchasing Power Parity (2) Bank Rate (3) Wage Rate
(4) Exchange Rate (5) None of these
Q.442. Prior approval (as also a licence) of RBI is required for opening_______.
(A) Personal banking branches
(B) Merchant banking branches
(C) Asset recovery branches
(1) Only A and C (2) Only B (3) All A,B and C (4) Only A (5) None of these
Q.443. Treasury bill is ___________
(1) Negotiable security (2) Non negotiable security
(3) No security at all (4) Quasi negotiable security
(5) None of these
Mahendra's GENERAL AWARENESS
Q.444. Banking Regulation Act 1949 does not apply to _________
(1) Nationalised Banks
(2) Foreign Banks having branches in india
(3) State Bank of India and its associates Banks
(4) Primary agricultural credit socities and co-operative land mortgage banks
(5) None of these
Q.445. The various means of Electronic Payment Systems Includes________
(A) Credit Card
(B) Debit Card
(C) ATM Card
(1!) Only A (2) Only B (3) Only C (4) All A,B & C (5) None of these
Q.446. Which of the following will replace the present tax structure like income tax, corporate tax etc from
next year ?
(1) GST (2) CENVAT (3) DTC (4) MODVAT (5) None of these
Q.447. What is the maximum amount of loan available to individuals on securities of demat shares ?
(1) `10 lacs (2) `15 lacs (3) `20 lacs (4) `25 lacs (5) None of these
Q.448. Which one of the following monitors the liberalized remittance scheme, through which resident
individuals may remit up to USD 200,000 per financial year for any permitted capital and current
account transactions or a combination of both ?
(1) Ministry of Finance (2) Ministry of Overseas Indian Affairs
(3) Department of Posts (4) Reserve Bank of India
(5) None of These
Q.449. What is the minimum period for which an NRE (Non Resident External) term deposit account can
be opened ?
(1) 12 months (2) 36 months (3) 60 months (4) 52 months (5) None of these
Q.450. Which of the following is not true about RTGS transaction ?
(1) The minimum amount to be remitted through RTGS is Rs. 2 lakh.
(2) There is no ceiling for RTGS transactions.
(3) Transactions are settled in batches
(4) Inward transactions are free
(5) None of these
Q.451. If a credit card is lost, the card member should call up the Customer Helpline number and he says
his request for deactivating the card. This is called-
(1) Hotlisting (2) Crosslisting (3) Shortlisting (4) Blocklisting (5) None of these.
Q.452. A cheque which is issued today must be presented before at bank for payment within a stipulated
period. After expiry of that period, no payment will be made and it is then called .
(1) Mutilated Cheque (2) Stale cheque (3) Crossed cheque
(4) Open cheque (5) None of these
Q.453. Which of the following is not true about NPA in banks ?
(1) It is usually a loan that is not producing income
(2) It is created If a borrower misses paying his equated monthly instalment (EMI) for 90 days.
(3) High NPA are a sign of good financial health.
(4) Asset Reconstruction Companies (ARC's)buy bank NPA at a discount along with the right to
recover the money.
(5) None of these
Q.454. What is the maximum population limit for a commercial bank branch to be declared as rural branch?
(1) 1 lac (2) 2 lac (3) 4 lac (4) 5 lac (5) None of these.

GENERAL AWARENESS Mahendra's


Q.455. What does Current Ratio mean?
(1) companys ability to pay back its long-term liabilities
(2) it is the ratio of net debt to total liabilities
(3) it is the ratio of liquid assets to current liabilities
(4) companys ability to pay back its short-term liabilities
(5) None of these
Q.456. Interest spread is a commonly used term in banking. What does it describes ?
(1) The sum total of discount and interest from loans.
(2) aggregate deposits plus aggregate advances.
(3) total interest earned over total interest expended.
(4) total interest earned less the non-interest expenses.
(5) None of these
Q.457. Consider the following statements-
(A) The Kisan Credit Card (KCC) Scheme was introduced in August 2000.
(B) Allied and non-farm activities are also covered under the Scheme.
(C) Validity period of the KCC is 3 years.
Which of the statements given above is/are true ?
(1) Only A (2) Only B (3) Only C
(4) All A, B and C (5) None of these
Q.458. Recently RBI has directed Banks to issue prepaid Payment Instruments to listed Corporates. Which
of the following are examples of prepaid Payment Instruments-
(A) smart cards (B) paper vouchers
(C) mobile accounts (D) internet wallets
(1) Only A and B (2) Only B and C (3) Only A, B and D
(4) Only A, C and D (5) All A, B, C and D
Q.459. Which of the following Private sector Banks has launched personal loan Swift?
(1) Axis Bank (2) ICICI Bank (3) HDFC Bank
(4) IDBI Bank (5) None of these
Q.460. The crop insurance scheme is being implemented at which among the following levels under the
National Agricultural Insurance Scheme (NAIS)?
(1) Village (2) Panchayat (3) District (4) State (5) None of these
Q.461. "Honours your Trust" is a Tag line of which bank ?
(1) Andhra Bank (2) Vijaya Bank (3) Dena Bank
(4) UCO Bank (5) None of these
Q.462. Which among the following is / are the main objective(s) of National Green Crops ?
(1) Educate youngsters about environment and eco-systems.
(2) Prosecute and Punish the violators of Environment laws.
(3) Promote Green Products.
(4) All of above
(5) None of these
Q.463. The Finance Ministry released the draft national public-private partnership (PPP) policy. Which of
the following facts given below in this regard are NOT TRUE?
(A) PPP is aimed at creating a framework for implementing infrastructure projects across sectors.
(B) It addresses the issues concerning definition of various terms and also processes so that a
clear and consistent position can be adopted by stake holders, including centre, states and
private investors.
(C) The government is envisaging an investment of USD 7 trillion in infrastructure sector during
the 12th Five-Year Plan (2012-17).
(1) Only C (2) Only A and B (3) Only B and C (4) All of these (5) None of these

Mahendra's GENERAL AWARENESS


Q.464. Which of the following organizations is created specially for providing credit to small and medium
enterprises in India ?
(1) NABARD (2) SIDBI (3) ECGC
(4) AMFI (5) None of these
Q.465. What is full form ofNBFCas used in financial sector ?
(1) New Banking Finance Corporation
(2) Non Banking Finance Company
(3) New Business Formation Company
(4) Net Banking & Finance Cooperative
(5) None of these
Q.466. Which of the following terms is NOT used in the field of Banking / Finance ?
(1) Goods & Services (2) Refund
(3) Tax Deducted at Source (4) Fundamental Duties of a citizen
(5) Relief to exporters
Q.467. What does the letter V denotes in the term VAT as used in banking world ?
(1) Valuable (2) Various (3) Vendor (4) Very (5) Value
Q.468. Which of the following is called the World Bank ?
(1) International Monetary Fund (2) Asian Development Bank
(3) World Trade Organisation (4) International Bank for Reconstruction and Development
(5) None of these
Q.469. We frequently read about ' MFI' in newspapers. What is the full form of this term, which is used in
financial sector ?
(1) Micro Finance Institutions (2) Micro Functional Issues
(3) Mutual Funds in India (4) Monetary Forum of India
(5) Monetary and Financial Institutes
Q.470. With reference to Banking, The 15th day of a month is known as _________
(1) Banking Day (2) Customer's Day (3) Holiday
(4) Complaints Day (5) None of these
Q.471. 'A customer is a person who maintains an account with a Bank' This definition of the customer is as
per _____________.
(1) Banking Regulation Act,1949 (2) Negotiable Instrument Act,1881
(3) RBI Act,1934 (4) KYC guidelines of RBI
(5) None of these
Q.472. Which of the following cannot be endorsed ?
(1) Fixed deposit receipt (2) Bank draft (3) Promissory note
(4) Cheque (5) None of these
Q.473. Which of the following conducts supervision over Commercial Banks, Financial Institutions, NBFCs
and other Para - Banking financial institutions in India ?
(1) SEBI (2) Ministry of Finance
(3) Board for Financial Supervision, RBI (4) Ministry of Corporate Affairs
(5) None of these
Q.474. The Export - Import Bank of India was set up in _______
(1) April 1982 (2) January 1982 (3) April 1970 (4) July 1969 (5) None of these
Q.475. In India, the Real Time Gross Settlement has been implemented by ___________
(1) SEBI (2) IRDA (3) RBI
(4) Ministry of Finance (5) None of these

GENERAL AWARENESS Mahendra's


Q.476. The maximum no. of transactions by a customer by using ATM of other Bank is _________
(1) 8 per month (2) 5 per month (3) 2 per month
(4) 3 per month (5) None of these
Q.477. The full form of SPNS is __________
(1) Shared Payment Network System (2) Sharing Process Network System
(3) Shared Procedural Network System (4) Shortest Possible Networking System
(5) None of these
Q.478. The Tagline "Where every individual is committed" is associated with which of the following Banks ?
(1) Punjab National Bank (2) Oriental Bank of Commerce
(3) Indian Overseas Bank (4) Syndicate Bank
(5) None of these
Q.479. "Doha Development Round" is related with which of the following organization ?
(1) World Bank (2) World Trade Organisation
(3) International Monetary Fund (4) Asian Development Bank
(5) None of these
Q.480. The rate at which Banks borrow money from the Reserve Bank is called ___________.
(1) Repo rate (2) Reverse repo rate (3) Bank rate
(4) Base rate (5) None of these
Q.481. The Banking Ombudsman Scheme 2006 Covers _________
(A) Commercial banks
(B) Regional rural banks
(C) Scheduled Primary Co-operative Banks
(1) Only A (2) Only B (3) All A, B and C
(4) Only C (5) None of these
Q.482. In case of a demand draft is issued, the liability of the bank will be _________
(1)Demand liability (2) Time liability (3) Current liability
(4) Long term liability (5) None of these
Q.483. All commercial banks including branches of foreign banks functioning in India, local area banks and
regional rural banks are insured by the DICGC. What is the full form of DICGC ?
(1) Deposit Insurance and Credit Guarantee Company
(2) Deposit Insurance and Credit Guarantee Corporation
(3) Direct Insurance and Credit Guarantee Corporation
(4) Direct Insurance and Credit Guarantee Company
(5) None of these
Q.484. Name a Poverty Eradication Programme in which banks played the dominant role?
(1) IRDP (2) MNP (3) Rojgar Bima Yojana
(4) Jawahar Rojgar Yojana (5) None of these
Q.485. Which pair is not correct ?
(1) EXIM Bank - Financing for export-import
(2) RBI - Banker's bank
(3) IDBI - Industrial Finance
(4) FCI - Financial assistance to commercial institutions.
(5) All are correct
Q.486. The minimum paid-up capital requirement at present for new Private Bank in India is -------.
(1) ` 100 crore (2) ` 200 crore (3) ` 500 crore (4) ` 600 crore (5) None of these

Mahendra's GENERAL AWARENESS


Q.487. CRISIL ------
(1) Evaluates the credit documents of a company
(2) Provides financial assistance to industries
(3) Provides finance for rural areas
(4) Promotes exports
(5) Promotes imports
Q.488. According to Banking Regulation Act, RBI can fix SLR upto the ceiling of --------
(1) 40% (2) 50% (3) 60% (4) 75% (5) None of these
Q.489. India Development Bond was issued in 1991-92 by --------.
(1) SBI (2) RBI (3) UTI (4) NHB (5) None of these
(5) None of these
Q.490. Which of the following is not helpful in controlling money supply?
(1) Free Market Policy (2) CRR (3) Bank Rate
(4) Change in Margin Requirement (5) None of these
Q.491. The first public sector bank in India which obtained license for Internet Banking from RBI is ---
(1) Punjab National Bank (2) Oriental Bank of Commerce
(3) Corporation Bank (4) State Bank of India
(5) None of these
Q.492. Which of the following measures can check Inflation?
(1) Surplus Budgeting (2) Increase in direct taxation
(3) Curtailment in public expenditure (4) All of the above
(5) None of these
Q.493. According to the Banking Regulation Act, nationalized bank can release a share of total paid up
capital up to maximum ceiling of --------.
(1) 40% (2) 49% (3) 51% (4) 73% (5) 100%
Q.494. The RBI's methods of credit control may be broadly divided into ------
(1) Quantitative and Qualitative (2) Open and Close
(3) Monetary and Fiscal (4) Rural and Urban
(5) None of these
Q.495. Which one of the following is not a liability of commercial banks?
(1) Time deposits
(2) Borrowing from Central Bank
(3) Security holdings
(4) Demand deposits
(5) None of these
Q.496. The oldest Central Bank in the world is the ------
(1) Reserve Bank of India (2) Bank of England
(3) U S Federal Reserve (4) State Bank of India
(5) None of these
Q.497. General Insurance Corporation of India (GIC) was formed in ---------
(1) 1972 (2) 1973 (3) 1974 (4) 1975 (5) None of these
Q.498. 'Open Market Operations' mean sale or purchase in the open market of ------
(1) Foreign Exchange (2) Goods and Services (3) Gold Bullion
(4) Government Securities (5) None of these

GENERAL AWARENESS Mahendra's


Q.1.(3)
EXPLANATION
Punjab National Bank was established in concurrent list.
the year 1894. Q.29.(1) "Operational Risk" in the field of Banking is
Q.2.(4) Centralized Online Realtime Exchange risk of loss due to inadequate or failed
Q.3.(1) Q.4.(1) internal process.

Q.5.(2) In Open unemployment people are willing Q.30.(4) Code sharing is term related to Air
to work but they do not get it. Transport Industry.

Q.6.(3) Credit Policy (also called monetary policy) Q.31.(3) WADA is related to Dope Testing.
Q.7.(4) Book Keeping entry only. Q.32.(4) Bank Rate is a rate at which commercial
banks borrow long term funds from RBI.
Q.8.(5) United Stock Exchange will function in four
currency derivatives. Q.33.(4)

Q.9.(3) Central Statistical Organisation Q.34.(3) "World Economic Outlook" report is


released by International Monetary Fund.
Q.10.(1) GDP It is based upon survey on the different
Q.11.(2) Regulation of inter-state trade developing economies.
Q.12.(4) Never Q.35.(3) Money laundring is way of converting black
money to white money.
Q.13.(4) Salary & allowances of the Vice President
of India. Q.36.(3) Public Debt Act passed in 1944.
Q.14.(2) India got a new consumer price inflation Q.37.(2) In Indian economy m and m are. present.
1 3

measure, which could offer a much better It measures the total monetary resources
picture of price movements at the of the country.
consumer-level across the country. Q.38.(4) National Income is monetry value of all
Q.15.(1) FDI limit in Public Sector Bank is 20%. final goods and services of a
country.
Q.16.(4) It is 940 females over 1000 males in
census 2011 as compared to 933 females Q.39.(3) In India First Five year plans are formed by
in 2001 census. planning commission in 1951.
Q.17.(5) All of the above Q.40.(3) It is a process developed by SEBI for
applying to IPO. In ASBA, an IPO
Q.18.(2)
applicant's account doesn't get debited
Q.19.(1) Fist Five Year Plan until shares are alloted to him.
Q.20.(1) Call Money market is related to Share Q.41.(3) A debenture holder is a person or an
Market it is controlled by SEBI. organization that has lent money to a
Q.21.(2) Finance commission is included in Article company or firm in the form of debentures
280 of Indian constitution. or loan.
Q.22.(1) Q.42.(1) It provides post-retirement financial
security to over 2.73 lakh Gramin Dak
Q.23.(1) NAMA (Non Agriculture Market Access) is
Sevaks (GDS). The Department of Posts
the word is related to World Trade Organisation
has decided to deposit Rs.200 GDS per
Q.24.(3) SEBI is related to share market. U.K. Sinha month.
is Chairman of SEBI as on December 2011
Q.43.(2) NABARD was established on 12th July
Q.25.(1) National Planning Council is established in 1982 and owned by the Government of
1963. It is a non-constitutional body. India and Reserve Bank of India (RBI). It is
Q.26.(3) Post office do not sell stamp papers an apex development bank of the country
for supporting and promoting agriculture
Q.27.(3) Priority sector Advance is 10%. and rural development.
Q.28.(2) Economic planning comes under
Mahendra's GENERAL AWARENESS
Q.44.(2) Non-performing assets, also called non- Q.56.(3) This is to improve the present status of
performing loans, are loans, made by a women in agriculture and to enhance the
bank or finance company, on which opportunities for her empowerment and is
repayments or interest payments are not a sub component of National Rural
being made on time. Livelihood Mission.
Q.45.(4) The departments under finance ministry Q.57.(5) All are development banks.
are: Q.58.(5) Bank of Sweden Prize is another name for
(A) Economic Affairs Noble Prize for Economics.
(B) Expenditure Q.59.(4) Market Cost = Factor Cost +Advertisement
(C) Revenue Cost + Transportation Cost

(D) Financial Services Q.60.(5) In first phaze in the year 1969 - 14 banks
were nationalised with a paid up capital of
(E) Disinvestment Rs. 50 crores.
Q.46.(2) Aixs bank is a private sector bank in India.
Q.61.(2) The Banking Ombudsman, promoted by
Q.47.(3) IDR is an instrument in the form of a RBI provides speedy solutions to the
Depository Receipt created by the Indian grievances faced by the customers from
depository in India against the underlying various banks.
equity shares of the issuing company. Q.62.(1) Targated Growth rate is 8% in 12 th
Five
Q.48.(1) Income Recieved by public borrowing year plan.
Q.49.(1) Interbank Mobile Payment Service (IMPS) Q.63.(3) Gilt - Edged market means Market of
launch on 22nd November 2010 by Government Securities.
Shyamala Gopinath, DG RBI at Mumbai Q.64.(4) A unit-linked insurance plan (ULIP) is a
and this service is now available to the type of life insurance where the cash value
Indian public. IMPS offers interbank of a policy varies according to the current
electronic fund transfer service through net asset value of the underlying
mobile phones. investment assets.
Q.50.(1) National Security Depository limited. Q.65.(3) Currency devaluation takes place when
Q.51.(3) All are true. one country's currency is reduced in value
in comparison to other currencies.
Q.52.(5) Swabhiman is a nationwide programme on
financial inclusion. Q.66.(1) Foreign Exchange Management Act.
Q.53.(3) Badla is a carry forward system in stock Q.67.(4) Narrow Banking restrict banks to hold
trading. liquid and safe government bonds

Q-54.(3) A committee on capital account Q.68.(3) This exchange rate is used to determine an
convertibility was setup by the Reserve individual country's currency value relative
Bank of India (RBI) under the to the other major currencies in the index,
chairmanship of former RBI deputy as adjusted for the effects of inflation.
governor S.S. Tarapore to "lay the road Q.69.(4) All are true.
map" to capital account convertibility.
Q.70.(4) IRDA allows portability shifting of policies
Q-55.(5) Prime Minister, Dr. Manmohan Singh, from one insurer to another on same terms
inaugurated a call centre 'Udyami Helpline, from July 1, 2011.
which will give details about business Q.71.(3) Equity (finance), the value of an ownership
possibilities and loan facilities to i n t e r e s t i n p r o p e r t y, i n c l u d i n g
entrepreneurs keen on setting up small shareholders' equity in a business.
units.
Q.72.(1)
GENERAL AWARENESS Mahendra's
Q.73.(1) Khusro committee. Q.96.(1) Transparency in Lending Rates of Bank.
Q.74.(4) Non-Banking Financial Company, a Q.97.(2) Commercial papers are a money-market
company registered under the Companies security issued by large banks &
Act, 1956 of India, engaged in the business corporations to get among to meet short
of loans and advances, acquisition of term debt obligations.
shares, stock, bonds, debentures and Q.98.(2) The proposed bill is on the lines of Right to
securities issued by government or local Information Act.
authority, or other securities of a
Q.99.(4) Ablation is the process of transferring heat
marketable nature, leasing, hire-purchase, by removing material through melting,
insurance business, or chit business. vaporization or other erosive process.
Q.75.(4) Prevention of Money Laundering Act. Q.100.(4) Treasury Bills are the instruments of short
Q.76.(3) Bad debts occur when the debtor has term borrowings by the Central /State
declared bankruptcy, and if the bad debts Govt.
are recovered it will be considered as an Q.101.(2) Fiscal Policy.
income.
Q.102.(3) In 1980, 6 banks were nationalised with a
Q.77.(2) Epicenter is point on earth surface that is capital of 200 crores.
above hypocenter.
Q.103.(4) Net Demand and Time liabilites.
Q.78.(4) NABARD
Q.104.(5) Base year for all India Index of Industrial
Q.79.(2) Production is 2004-2005 with base Rs.100
Q.80.(3) Ministry of Finance Q.105.(4) Regulation of Inter state Tract.
Q.81.(2) LLP is a partnership in which all partners Q.106.(3)
have a limited liability.
Q.107.(3) The period of fourth plan was 1969 to
Q.82.(3) 1974.
Q.83.(5) 2009 - 2010 Q.108.(2) Indian Gold Council.
Q.84.(1) 2004 Q.109.(3)
Q.85.(2) 1998 Q.110.(1) It is one of the lending arms of World Bank
Q.86.(4) Cheap Money Group.
Q.87.(3) Federal Reserve is Central Bank of Q.111.(2) Branch Banking.
America Q.112.(3) Near money is a term used in economics to
Q.88.(3) Dr.Vijay Kelkar is the chairman of 13 th
describe highly liquid assets that can be
Finance commission. easily converted into cash.
Q.89.(3) RRB'S - includes Q.113.(1)
50% --- Central Govt. Q.114.(1) Amount of gold that each member country
15% --- State Govt. of IMF contributes as part of its
membership obligations to the fund and
35% --- Sponsor bank/Lead bank can borrow when need arises.
Q.90.(2) Asian Development Bank. Q.115.(3) It was launched with an aim of enhancing
Q.91.(4) farm profits, creating employment
opportunity, restoring soil fertility.
Q.92.(1) Liberalised Exchange Rate Management
System. Q.116.(1) Currency Notes.
Q.93.(4) Q.117.(3) Flare
Q.94.(4) It is a condition of high inflation in period of Q.118.(3) Employee Stock Option Plan
high unemployment Q.119.(3) Fiat money is money that has value only
Q.95.(2) Cash reserve ratio is a part of RBI's monetary because of government regulation or
policy and all other are the steps taken by law.
government in its fiscal policy

Mahendra's GENERAL AWARENESS


Q.120.(1) Unit banking refers to a single bank which Q.139.(3)
tender services and operates with out any Q.140.(4) An increase in the supply of money works
branches anywhere. both through lowering interest rates which
Q.121.(3) Pipette is related to chemistry. spurs investment, and through putting
Q.122.(3) Fixed Income Money Market and more money in the hands of consumers
Darivative Association making them feel wealthier and thus
stimulating spending.
Q.123.(4)
Q.141.(2) A bank, included in 2nd schedule of RBI
Q.124.(3) Soft Patch is a period of economic slow Act 1934
down in the middle of a larger trend of
economic growth. Q.142.(4) Moral suasion is a qualitative credit control
measure of the Reserve Bank of India
Q.125.(2)
Q.143.(3) S. Chakravarty.
Q.126.(1) RBI uses CRR either to drain excess
liquidity or to release funds needed for the Q.144.(3) The IMF pays interest on all holdings of
economy from time to time. Increase in SDRs cept in the special Drawing Account
CRR means that banks have less funds and charges interest at the same rate on
available and money is sucked out of allocations to participants.
circulation. Q.145.(1) The main objective of RRBs is to provide
Q.127.(3) Negotiated Dealing system is an electronic credit and other facilites particulary to the
platform for facilitating dealing in small and marginal farmers, agricultural
government securities and money market labourers, artisans and small entrepreneurs.
instruments. Q.146.(1) Deficit financing means increasing the
Q.128.(1) Quarter amount of money in circulation by printing
and pumping in more paper currency.
Q.129.(4) GDP is the money value of all final goods &
Q.147.(4) 1951
services produced during a financial year
&within the geographical boundary of a Q.148.(1) Acworth committee.
country. Q.149.(3) Money Market.
Q.130.(3) 2002 Q.150.(1)
Q.131.(4) BANKNET Q.151.(4) Primary Deficit = Fiscal Deficit - Interest
Q.132.(2) payment.
Q.152.(2) 1st nationalisation took place in 1969.
Q.133.(3) Gold standard is a monetary system in
which the standard economic unit of IInd nationalisation took place in 1980.
account is a fixed mass of gold. Q-153.(2) Time liabilities mean FDs, cash
Q.134.(4) Safe Deposit certificate,time liabilities portion of saving
Q.135.(4) The new issue market consists of bank deposits.
securities issued directly to the investors Q-154.(4) Subsidies & Interest payments called non-
through machanism called primany plan expenditure because provision for
market. these expenditures is not made in the five
Q.136.(4) Redeemable preference shares, 15 years year plans.
Q.137.(4) Since these are ceiling rates, banks are Q-155.(3) Industrial manufacturing and Industrial
free to decide the rate of interest to be construction comes under secondary
charged. sector.
Q.138.(4) Paid up capital is the total amount of Q-156.(2) Sales Tax. Because the Tax is not borne by
shareholder capital that has been paid in the producer on whom it is imposed , but is
full by shareholders passed on to the consumer.

GENERAL AWARENESS Mahendra's


Q-157.(2) The 7 schedule of the Indian Constitution
th country. It is one of the most important duty
deals with Banking. because it hampers illegal import and
export of goods.
Q-158.(3) Monetary and credit policy.
Q.178.(4) With aim of giving a fillip to development
Q-159.(3) A soft currency indicates a currency which schemes in triable and backward regions
is expected to depriciate against other affected by Naxal violence, Govt. of India
currencies. approved the Integrated action plan in 60
seclected triable and backward districts.
Q-160.(5) Digital signature is Encrypted signature for
providing security. Q.179.(3) Customs Duty and Corporation Tax
belongs to the union government alone.
Q-161.(4) Transfer payment are those transactions
Q.180.(1) 1993
which are not related with the production of
goods and services. These are not Q.181.(2) Reserve Bank of India.
included in national in come. Q.182.(4) The National Housing Bank Launched
RESIDEX Index for tracking prices of
Q-162.(1) Reduction in CRR increases the liquidity
residential properties in India.
position with in Indian banks.
Q.183.(2) Mumbai
Q-163.(1) 1988
Q.184.(4) Allowing business units to close down.
Q-164.(2) Primarily that of a debtor and a creaditor
Q.185.(2) A decrease in loans.
Q-165.(3) In the form of Term Deposits only FCNR Q.186.(4) Commercial borrowing is not a component
accounts can be opened and maintained. of foreign aid of a country.
Q-166.(3) Current Account includes Trade Account Q.187.(2) Lending to agriculture, small scale
and Invisibles. industries and the weaker sections of the
society comes under priority sector lending
Q-167.(4) Joint supply refers to a product or process of commercial banks in India.
that can yield two or more outputs.
Common example occur within the live Q.188.(1) Increased.
stock industry, sheep can be utilized for Q.189.(4)
meat, wool and sheep skin. Q.190.(2)
Q-168.(3) More than one creditors coming together Q.191.(5) Encryption is a process that is used to
to provide single loan. ensure the privacy and security of a
Q.169.(3) An economy that does not interact with any person's confidential financial information
other economy Q.192.(3)
Q.170.(1) Expansionist open market operations Q.193.(1)
policy focuses on increasing liquidity in the
Q.194.(5) All 1, 2 and 3 are the limitations and along
common market.
with them cash reserve ratio, Nature of
Q.171.(4) Moral Suasion is qualitative measure of business conditions in the economy,
control of RBI's monetary policy. sound securities, liquidity preference and
Q.172.(4) In GDP of London. monetary policy of Central Bank are the
limitations on credit creation
Q.173.(1)
Q.195.(3)
Q.174.(2)
Q.196.(2)
Q.175.(3)
Q.197.(1) It aims at 50 food parks to minimize
Q.176.(3) Formulation of the Monetary policy is the
damage to huge amount of fruits and
sole responsibility of the central bank of the
vegetables in the absence of wider
country.
network of food processing units
Q.177.(3) The custom duty is imposed on the
imported and exported goods in the Q.198.(3) Q.199.(3) Q.200.(1)

Mahendra's GENERAL AWARENESS


Q.201.(2) Legal Tender Money is a type of money Q.218.(1)
protected by law. Q.219.(1) Foreign currency & securities held by the
Q.202.(4) banks and corporate bodies do not form a
Q.203.(3) RBI is the Central Bank which controls the part of India's foreign exchange reserves.
banks in the Economy. Q.220.(3) Gains results from fact that a higher total
Q.204.(5) It is General crossing. output is allocated between two countries
& there is increased welfare in each
Q.205.(4) Working Capital is a measure of both a country.
company's efficiency and its short term
financial health. Q.221.(3) The Reserve Bank of India has been
intervening in the foreign exchange market
Q.206.(4) A FER is a type of exchange rate regime of India and buying dollors to prevent the
wherein currency's value is allowed to rupee strengthening further, thereby acting
fluctuate according to the foreign as disincentive to export growth.
exchange market
Q.222.(2) Non-performing assets of Commercial
Q.207.(2) Banks means their loans for which
Q.208.(4) CD was introduced to enable banking interest/instalment has remained unpaid
system to mobilize bulk deposits from the after due date.
market, which they can have at Q.223.(2) In Real Time Gross settlement system the
competitive rates of interest. They can be settlement of funds is not revocable.
issued by all scheduled commercial
banks and all India FI's with their umbrella Q.224.(3) The definition of Banking is given in the
limits. Banking Regulation Act, 1949.

Q.209.(5) RBI increases the minimum Statutory Q.225.(3) In FCNR accounts the payment of Interest
Cash Reserve Ratio of Commercial banks is effected in same currency in which
when the economy is experiencing deposit stands.
inflation and when the central bank aims Q.226.(1) Commercial paper is a money - market
credit contraction. security issued by large commercial banks
Q.210.(1) to meet short term debt obligations.

Q.211.(3) Assets that can be quickly convertible into Q.227.(3) The methods used by the RBI for credit
cash. control are qualitative and quantitative.

Q.212.(4) Q.228.(1) Increase in Bank rate is generally followed


by an increase in market rates of Interest.
Q.213.(2) OMO is the buying and selling of
Government bonds on the open market by Q.229.(2) Commercial Banks on the basis of their
a central bank. primary deposits mainly creates credit.

Q.214.(1) Q.230.(3) Machinery is the least liquid asset.

Q.215.(3) Treasury bills are issued by Government & Q.231.(2) The most important component of the
sold by RBI in form of small bonds to liabilities of Commercial Banks in India is
banks. Time Deposits.

Q.216.(5) Bill of Exchange, bond, equity shares, Q.232.(1) An increase in foreign exchange assets of
saving deposit, with commercial banks, Reserve Bank of India means increase in
bankers acceptances, cash surrender high powered money.
values of life insurance policies, etc are Q.233.(2) Credit Deposit Ratio is not a method of
examples or considered as near money. credit control.
Q.217.(2) Forced savings are when consumers are Q.234.(2) The Largest Nationalised Commercial
not allowed to spend all of their income Bank in India is State Bank of India.
upon current consumption either self Q.235.(2) Reserve Money includes currency with the
imposed or imposed by contract. public and Cash with Banks.
GENERAL AWARENESS Mahendra's
Q.236.(4) Regulation of financial institutions is not an Q.254.(3) Creation of credit is a function performed
objective fiscal policy. by the commercial Banks in a country.
Q.237.(3) Education is not included in Infrastructure Q.255.(2) The Reserve Bank of India follows
in India. Minimum Reserve System of Note Issue.
Q.238.(3) The National Agricultural Insurance Q.256.(4) Free Trade Area means a group of
Scheme was started by the government of countries which abolished all Tariff and
India in 1999. quantitative restriction between
Q.239.(1) Interest on national debt is included in themselves.
GNP. Q.257.(3) Export
Q.240.(4) Disinvestment of equity in the public sector Q.258.(1)
is not a measure of globalization. Q.259.(2) Teaser loan are those loan in which rate of
Q.241.(3) Household sector contributes most to the interest in the initial period is less and goes
savings in India. up subsequently.
Q.242.(3) The State Bank of India was established on Q.260.(3) International Development Asociation
the recommendation of All India rural credit which is world Bank affiliate is connsidered
survey headed by Shri A.D. Gorewala. as 'soft loan' window.
Q.243.(2) Money Supply, M1 includes Currency + Q.261.(2) Wariable reserve ratio open market
Demand Deposits with Banks + Other operations are Instrument monetary policy
Deposits with RBI. Q.262.(2) MERA (Small & medium enterprises rating
Q.244.(3) Loans from central Government has been agency) was established in 2005.
the most important source of financing the Q.263.(2) Bank rate policy is the component of
state Government's Gross Fiscal Deficit in Monetary policy
India.
Q.264.(3)
Q.245.(1) The system of Ad-hoc Treasury Bills in
India was replaced by Ways and Means Q.265.(4) If the RBI sells securities in the market the
Advances with effect from 1 April, 1997. Bank deposits will deadline

Q.246.(2) In the context of liberalisation and Q.266.(2) The majority of central Government
globalisation planning has an indicative Q.267.(2) The central Bank can decrease the Bank
role to play in providing safety net for the credit component of the money supply by
poor. increasing the Bank rate.
Q.247.(4) Corporation Tax is levied by the union and Q.268.(3) Parallel Economy is also referred as Black
belongs to it exclusively. Economy.
Q.248.(4) Treasury Bills is an Example of Near Q.269.(2) Oudh commercial Bank was the first
Money. commercial Bank of limited liability
Q.249.(2) Mutual Fund manged by Indians.

Q.250.(4) Expansion is not an objective of Bank's Q.270.(3) A letter of credit is a letter from a Bank,
portfolio management. guarantceing that a by yea's payment to a
seller will be received on time and for the
Q.251.(2) A Bank which controls credit is called correct amount.
Central Bank.
Q.271.(2) Time liabilitiy are those liabilities which
Q.252.(4) When the government imposes physical Bank have to pay after specific time
and monetary controls to check open period. Fixed deposits, cash certificate,
inflation it is known as suppressed time liablities portion of Saving bank
inflation. deposits come under Time Liabilites.
Q.253.(3) The cause of deflation is lack of money Q.272.(4) Regional Rural Banks are classfied under
supply as compared to supply of goods public sector Banks
and services.
Mahendra's GENERAL AWARENESS
Q.273.(3) Government of India deades the currency Q.286.(3) The ability of a Commercial Bank to
denominatio value. increase its loans and Investment depends
on its excess reserves.
Q.274.(1) The country's first ever socio-economic
and caste census was started from Q.287.(2) Economic Development is a continuous
Hezamara in Tripura state. and long term process.
Q.275.(4) Spandana is India's leading microfinance Q.288.(5) Microfinance is a type of banking service
institutions. It provides financial service that is provided to unemployed or low-
towards lower income customers, income individuals or groups who would
primarily women. otherwise have no other means of gaining
financial services. Microfinance provides
Q.276.(3) The Central Bank increases the minimum
low income people an opportunity to
Statutory cash reserve ratio of the
become self-sufficient by providing a
Commercial Banks when the economy is
means of saving money, borrowing money
experiencing Inflation and the Central
and insurance.Interest rates charged by
Bank aims credit contraction.
microfinance institutions are generally
Q.277.(4) The primary objective of selective credit lower than those offered by normal banks,
control is to influence allocation of credit some opponents of this concept condemn
among different borrowers and users. microfinance operations for making profits
Q.278.(2) All the given three quantitative measures off of the poor.
i.e. changing the bank rate, open market Q.289.(1)
operations and increasing the cash-
Q.290.(4) Cheque Book is not a value added service
reserve ratio are taken by RBI to control
offered by a bank.
inflation.
Q.291.(1)
Q.279.(2) Bank rate refers to the rate at which
Central Bank rediscount bills of exchange Q.292.(5) PURA stands for Provision of Urban
of Commercial Banks. Amenities in Rural Areas
Q.280.(1) Deposits with the Central Bank and cash in Q.293.(2) Banking Sector comes under Service
hand reduce the money available for credit Sector.
creation. Q.294.(2) The acronym 'RTGS' stands for Real Time
Q.281.(1) Short term commercial borrowings is Gross Settlement, which can be defined as
included in the capital account of balance the continuous (real-time) settlement of
of payment. funds transfers individually on an order by
order basis (without netting).
Q.282.(1) Frequent changes in the Bank rate leads to
the expansion and contraction in the flow Q.295.(1) The Securitisation and Reconstruction of
of money supply, thats why RBI placed the Financial Assets And Enforcement of
greatest reliance on the Bank rate during Security Interest Act, 2002 (SARFAESI
the last ten years. Act) is an Act to regulate securitisation and
reconstruction of financial assets and
Q.283.(2) Foreign Exchange reserves includes the
enforcement of security interest and for
following components-
matters connected therewith or incidental
1- Gold thereto.
2- Special Drawing Rights (SDRs) Q.296.(1) Q.297.(2) Q.298.(4)
3- Foreign - currency assets Q.299.(2) CIBIL
4- Reserve Tranche with I.M.F. Q.300.(1) Banking sector
Q.284.(2) Increasing Trend. Q.301.(2) ASEAN
Q.285.(3) In case of closed economy GNP and GDP Q.302.(2) NABARD
would be same because there is no net
Q.303.(3) The Prime Minister
income earned from abroad.
GENERAL AWARENESS Mahendra's
Q.304.(1) Q.305.(2) Q.306.(3) Q-325.(4) Small loans provided by the banks to very
Q.307.(1) poor families is known as Micro finance.
Q.308.(2) A,AA+ & AAA are credit ratings Q-326.(2) "One Family One Bank" is the tagline of
Q.309.(1) High powered money is the Bank's Bank of Maharashtra.
reserves at Central Bank Q-327.(2) Co-operative movement has been highly
Q.310.(1) A significant decrease in the demand for successful in the Milk production.
loans will force banks to adjust their
Q-328.(1) UNDP uses Human development Index for
portfolios.
measurement of the quality of people's life.
Q.311.(3) The process of credit creation has the
effect of increasing the supply of money. Q-329.(1) CAPART was set up in 1986 to promote
new technological inputs rural sector.
Q.312.(4) Controlling credit expansion is the function
of central Bank Q.330.(4) Nabard has stabilized core banking
infrastructure of insuring prosperity and
Q.313.(3) M is the Broader measure of money
3
progress in integrated development of
supply M is Narrow money.
1
Rural Area.
Q.314.(2) RBI has helped to finance India's foreign
Q.331.(3) Open market operation" controlled by RBI
trade through EXIM Bank.
Q.332.(2) Only 2
Q.315.(4) A Maharatna Company can invest Rs 5000
crore independently for a project. Q.333.(2) Q.334.(2) Q.335.(3)
Q.316.(3) Tier -1 capital is the core measure of a Q.336.(1) Q.337.(4) Q.338.(4)
Bank's Financial strength from a Q.339.(1) Q.340.(3) Q.341.(1)
regulatoris point of view.
Q.342.(4) Q.343.(1) Q.344.(5)
Q.317.(3) Swabhiman Scheme is related with
Q.345.(1) Q.346.(1) Q.347.(3)
Financial Inclusion.
Q.348.(1) Q.349.(4) Q.350.(1)
Q.318.(3) Provision coverage ratio refers to the
percentage of the loan amount that the Q.351.(4)
Bank has set aside as provisions to meet Q.352.(4) The minimum required capital to set up a
an eventuality. bank by a corporate entity is set at Rs.500
Q-319.(3) Goods and Services Tax will replace VAT. crore in comparison to the present
requirement of Rs. 300 crore.
Q-320.(4) Due to Increasing overhead cost of tax
Q.353.(4) The economy of India now a days facing
collection and the lack of political will, high inflation and high food and commodity
Agricultural Taxation in India is difficult. problems.
Q-321.(1) NABARD does not operate the Kisan Q.354.(2) The primary monetary policy technique
Credit Card scheme. employed by the Reserve Bank of India is
open market operations.
Q-322.(3) The sources of Non - Tax Revenue are -
Q.355.(3) When a Bank writes off a loan as bad, its
(1) Hospital fee on medical facilitics
total assets and capital decreases by that
(2) Entry fee on museums amount.
(3) Income from Radio & T.V. Broadcasting Q.356.(2) RBI maintains the cash reserve ratio
(CRR) which at present is 5.5%.
Q-323.(2) CBS stands for Core Banking Solution.
Q.357.(2) Channel Financing is the mechanism
Q-324.(2) Agriculture is the backbone of Indian through which a Bank/Financial institution
economy as it employed 52% of the total meets the various funds related
workforce. requirements along the supply chain at the
suppliers end.
Mahendra's GENERAL AWARENESS
Q.358.(1) Invisible export means export of services Q.374.(3) These currency swaps help to limit or
like consulting services, shipping services, manage exposure to fluctuations in
tourism etc. interest rates
Q.359.(2) Loan recovery rules is not the part of Q.375.(3) The third of the Basel Accords was
prudential guidelines issued by Reserve developed in a response to the
Bank of India. deficiencies in financial regulation
revealed by the global financial crisis.
Q.360.(2) A crossed cheque is a cheque which is
payable only through a collecting banker Q.376.(2) M2 includes the amount of money in
and not directly at the counter of the bank. circulation in notes and coin plus non-
interest-bearing bank deposits, building-
Q.361.(2) A bank draft is traditionally a cheque drawn
society deposits, and National Savings
on a bank's funds
accounts.
Q.362.(3) Near money is highly liquid assets which
Q.377.(3) Contractual savings are in the form of
are not cash but can easily be converted
regular payments into long-term
into cash, such as bank deposits and
investments, such as pension schemes
Treasury Bills. Similar to cash equivalents.
Q.378.(2)
Q.363.(3) Due to steep reduction in direct taxes the
liquidity with people will be more, Q.379.(1) A foreign bank is a bank with head office
subsequently, the aggregate demand will outside the country in which it is located.
become higher. Due to rising demand Q.380.(1)
more and more people will be employed so
that production can be maximized. Q.381.(1)
Eventually, this increase in output will Q.382.(4) Statutory Liquidity Ratio or SLR refers to
become smaller and the price of goods will the amount that all banks require
rise due to capacity constraints. maintaining in cash or in the form of Gold or
Q.364.(1) RBI acts as the lender of the last resort by approved securities.
providing rediscount facilities to scheduled Q.383.(1) Excess liquidity in market
banks. Q.384.(2) Good people to banks with
Q.365.(4) Q.385.(1) CSO
Q.366.(5) Imperial Bank was formed after merging Q.386.(2) 1 Jan 1949
st

Bank of Bengal, Bank of Bombay and Bank


of Madras. Q.387.(1) 2 lakh
Q.367.(4) ICICI Bank has launched Easy Cash Card. Q.388.(3)
With this card, customers can load a Q.389.(3) Custom duty
personal loan up to Rs. 50,000 and can
Q.390.(4) Q.391.(5) Q.392.(5)
use it like a debit card to access money
instantly. Q.393.(3) Q.394.(1) Q.395.(3)
Q.368.(4) RBI has formulated a scheme called Q.396.(1) Q.397.(2) Q.398.(3)
"Protected Disclosures Scheme for Private Q.399.(5)
Sector and Foreign Banks". This move
Q.400.(2) Nuclear Disarmament
could prevent IPO scams in banks.
Q.401.(2)
Q.369.(2) United Nations launched a 'Decade of
Action for Road Safety' with the aim of Q.402.(1) 35%, the share in RRB by central Govt is
stabilizing and then reducing global road 50% & the state govt. is 15%
deaths by 2020. Q.403.(1) Discovery of oil & Natural has National
Q.370.(4) External Commercial Borrowings is an exploration liscensening policy.
instrument used in India to facilitate the Q.404.(1) 2004 - 05
access to foreign money by Indian
corporations and PSUs. Q.405.(2) Inflation, RBI increases repo rate to conrol
the cxcess flow of money in market (ie)
Q.371.(2) Q.372.(1) Q.373.(5) Inflation
GENERAL AWARENESS Mahendra's
Q.406.(1) Q.407.(4) re-load vouchers for mobiles and loan
Q.408.(3) Moral Suasion is Persuasion of banks to account enquiry.
adhere to central bank desires Q.425.(2) An EEFC (Exchange Earner's Foreign
Q.409.(4) PLR Currency) account can be held only in the
form of current account. No interest is
Q.410.(2) Only B payable on EEFC accounts.
Q.411.(1) 4th sept 1993 Q.426.(1) The full form of ADF is Automated Data
Q.412.(4) Public Account Deposit is not accepted by Flow. It seeks to ensure submission of
commercial banks. correct and consistent data from the banks
straight from their systems to RBI without
Q.413.(4) Production Method
any mannual intervention.
Q.414.(3) 16 PSEs(Public sector Enterprises)
Q.427.(1) A & C
Q.415.(2) B. Ashok
Q.428.(4) Indore
Q.416.(3) 188
Q.429.(2) Senior Citizen
Q.417.(4) As per RBI Governor, Inflation and
Q.430.(2) FEMA
supportive recovery of economy is a major
challenge to its monetary policy. Q.431.(4) Only B
Q.418.(2) The Reserve Bank of India has decided to Q.432.(3) Austerity Budget
introduced Rs. 10 polymer/plastic notes. Q.433.(3) Open market policy is not helpful in
Q.419.(3) Residence proof, latest salary slip and last controlling the money supply
six months bank statements are generally Q.434.(5) Piggy Banking is a form of "Small Savings
sought for housing loan approval. Bank" popular among the poor or children.
Q.420.(1) Loss making banks can make donations Q.435.(4) Overdraft occur when bank customers
up to Rs. 5 lakh only in a financial year. withdraw cash from their account, with the
Q.421.(2) The basic banking services of a bank balance going below zero. An overdraft is
includes services relating to deposit basically a form of credit extended by a
accounts, remittance facilities, collection creditor when the account balance
facilities. reaches zero. Overdrafts allow bank
clients to withdraw money even when
Q.422.(4) Primary co-operative societies are not
there are no funds in the account.
insured by the DICGC. In the event of a
bank failure DICGC protects bank deposits Q.436.(2) Bridge financing Also known as gap
that are payable in India. financing. Bridge financing begins at the
end of the time period of the first loan and
Q.423.(1) FDI is prohibited under the government
ends with the start of the time period of the
route as well as the automatic route in the
second loan, thereby bridging the gap
following sectors:-
between two loans.
- Retail Trading (Except single brand
Q.437.(1) A very large loan extended by a group of
product retailing)
small banks to a single borrower,
- Nidhi Company especially corporate borrowers is known
- Business of Chit Fund as syndicated loan.In most cases of
syndicated loans, there is a lead bank,
- Gambling and Betting
which provides a part of the loan and
- Lottery Business syndicates the balance amount to other
- Atomic Energy banks.

Q.424.(4) The other services available at ATMs Q.438.(2) The public debt office function under the
includes, account information, purchase of control of reserve bank of india..

Mahendra's GENERAL AWARENESS


Q.439.(2) Generally, any asset that can be converted continuous (real-time) settlement of funds
without loss in value or penalty within transfers individually on an order by order
twenty days is considered a liquid asset. basis, while NEFT settles transactions in
Liquid assets include money market fund batches.
shares, government securities, mutual Q.451.(1) If a credit card is lost, the card member
funds, and the cash value of a life should call up the Customer Helpline
insurance policy. Tax refunds and the number and give his request for hotlisting
balances of trust funds are often the card. Hot listing the card means
considered liquid assets. Goodwill, deactivating the card for any future
Trademarks, copyrights or patents, transactions.
Certificates of Deposits etc are non-liquid Q.452.(2) A cheque which is issued today must be
assets. presented before at bank for payment
Q.440.(3) The speculative capital flows are called within a stipulated period. After expiry of
"hot money" because they can move very that period, no payment will be made and it
quickly in and out of markets, potentially is then called stale cheque.
leading to market instability.
Q.453.(3) High NPAs are a sign of bad financial
Q.441.(1) When the exchange rate is adjusted so health of banks.
that an identical good in two different
Q.454.(1) 1 lac
countries has the same price when
expressed in the same currency, it is called Q.455.(4) Current Ratio is a liquidity ratio that
purchasing power parity measures a companys ability to pay short-
Q.442.(3) Prior approval (as also a licence) of RBI is term obligations. It is a ratio of current
required for opening Personal banking assets to current liabilities. It is also known
branches, Merchant banking branches as liquidity ratio, cash asset ratio and
and Asset recovery branches cash ratio.
Q.443.(1) Treasury bill is Negotiable security Q.456.(3) Interest spread describes the excess of
total interest earned over total interest
Q.444.(4) Banking Regulation Act 1949 does not
expended.
apply to primary agricultural credit
societies and co-operative land mortgage Q.457.(2) The Kisan Credit Card (KCC) Scheme was
banks. introduced in August 1998. Validity of the
Q.445.(4) The various means of Electronic Payment KCC has been extended from 3 years to 5
Systems Includes Credit Card, Debit Card years.
and ATM Card Q.458.(5) The Prepaid instruments can be issued as
Q.446.(3) Direct Taxes Code (DTC) will replace the smart cards, magnetic stripe cards,
multitude of direct taxes like income tax , internet accounts, internet wallets, mobile
corporate tax, wealth tax etc. from 2012, accounts, mobile wallets, paper vouchers
while GST will replace the indirect taxes and any such instruments which can be
like VAT, and service tax. used to access the prepaid amount
Q.447.(3) Amount of loan available to individuals on (collectively called Payment Instruments).
securities of demat shares is Minimum Rs. Q.459.(3) Private sector lender HDFC Bank has
1 lac, Maximum Rs.20 lacs. launched personal loan Swift.
Q.448.(4) The Reserve Bank of India monitors the Q.460.(3) District
liberalized remittance scheme, through
Q.461.(3)
which resident individuals may remit up to
USD 200,000 per financial year for any Q.462.(1) Educate youngsters about environment
permitted capital and current account and ecosystems is the main objective of
transactions or a combination of both. National cadet corps.
Q.449.(1) The minimum period for which an NRE Q.463.(1) Only C
term deposit can be opened is 12 months Q.464.(2) SIDBI is created specially for providing
and the maximum period is 60 months. credit to small and medium enterprises in
Q.450.(3) RTGS stands for Real Time Gross India
Settlement, which can be defined as the
GENERAL AWARENESS Mahendra's
Q.465.(2) The full form of 'NBFC' is Non - Banking Demand liabilities are the liabilities which
Finance Company must be met on demand.
Q.466.(4) Term Fundamental duties of a citizen is Q.483.(2) The Full form of DICGC is Deposit
NOT used in the field of Banking / Finance Insurance And Credit Guarantee
Corporation .
Q.467.(5) The letter 'V' denote in the term 'VAT' is
value. The full form of VAT is Value Added Q.484.(1) The Integrated Rural Development
Tax. Programme (IRDP) is a rural development
program of the Government of India
Q.468.(4) International Bank for Reconstruction and
launched in Financial Year 1978 and
Development is called the World Bank.
extended throughout India by 1980.
Q.469.(1) The full form of the term MFI is Micro
Q.485.(4) Food Corporation of India (FCI) was
Finance Institutions
established with the objective of fair
Q.470.(2) With reference to banking, The 15 day of a
th
distribution of food grains and to bring
month is known as customer's day. about stability in the prices in India.
Q.471.(4) As per KYC guidelines of RBI 'A customer Q.486.(3) The minimum paid-up capital requirement
is a person who maintains an account with at present for new Private Bank in India is
a Bank'. Rs. 500 crore.
Q.472.(1) Fixed deposit receipt cannot be endorsed. Q.487.(1) C r e d i t R a t i n g a n d I n f o r m a t i o n
Services of India Ltd (CRISIL) a
Q.473.(3) Board for Financial supervision, RBI,
global analytical company providing
conducts supervision over Commercial
ratings, research, and risk and policy
Banks, Financial Institutions, NBFCs and
advisory services.
other Para - banking financial institutions in
India. Q.488.(1) According to Banking Regulation Act, RBI
can fix SLR upto the ceiling of 40%.
Q.474.(2) The Export - Import Bank of India was set
up in 1982 under the Export - Import Bank Q.489.(1) India Development Bond was issued in
of India Act 1981. 1991-92 by SBI.
Q.475.(3) In India, 'The Real Time Gross Settlement' Q.490.(1) Free Market Policy is not helpful in
has been implemented by Reserve Bank controlling money supply.
of India.
Q.491.(1) The first public sector bank in India which
Q.476.(4) The maximum no of transactions by a obtained license for Internet Banking from
customer by using ATM of other Bank is 3 RBI is Punjab National Bank.
per month.
Q.492.(4)
Q.477.(1) The full form of SPNS is Shared Payment
Q.493.(1) According to the Banking Regulation Act,
Network System.
nationalized bank can release a share of
Q.478.(2) The tagline "Where every individual is total paid up capital up to maximum ceiling
committed" is associated with Oriental of 49%.
Bank of Commerce.
Q.494.(1) The RBI's methods of credit control may be
Q.479.(2) The Doha Development Round is the broadly divided into Quantitative and
current trade - negotiation round of the Qualitative.
World Trade Organisation which
Q.495.(3) Security holdings is not a liability of
commenced in November 2001.
commercial banks.
Q.480.(1) The rate at which Banks borrow money
Q.496.(2) The oldest Central Bank in the world is the
from the Reserve Bank is called Repo rate
Bank of England.
Q.481.(3) The Banking Ombudsman Scheme 2006
Q.497.(1) General Insurance Corporation of India
covers Commercial Banks, Regional Rural
(GIC) was formed in1972.
Banks and Scheduled Primary Co-
Operative Banks. Q.498.(4) 'Open Market Operations' mean sale or
purchase of Government securities in the
Q.482.(1) When a demand draft is issued, the liability
open market
of the bank will be demand liability.

Mahendra's GENERAL AWARENESS

Anda mungkin juga menyukai